Top Banner
PATHOMORPHOLOGY GENERAL 1. Histological examination of a 40 yo. man's thymus revealed decreased share of parenchymatous gland elements, increased share of adipose and loose connective tissue, its enrichment with thymus bodies. The organ's mass was unchanged. What phenomenon is it? A. Age involution B. Accidental involution C. Hypotrophy D. Atrophy E. Dystrophy 4. A 9 m.o. child has delayed dentition, it is also out of order. Upper jaw configuration is horizontal ("high"palate); microscopically irregular mineralization of tooth enamel, wrinkled enamel prisms, some of them are vacuolized. Predentin zone is extended; there are solitary denticles. What disease is it? A. Early rickets B. Hypervitaminosis D C. Late rickets D. Gout E. Osteomalacia 6. A patient ill with diabetes mellitus felt acute pain in his right foot. Objectively: foot thumb is black, foot tissues are edematous, there are foci of epidermis desquamation, stinking discharges. What clinicopathological form of necrosis is it? A. Infarction B. Sequestrum C. Dry gangrene D. Bedsore E. Moist gangrene 8. Autopsy of a man who died from influenza revealed that his heart was slightly enlarged, pastous, myocardium was dull and had specks. Microscopical examination of myocardium revealed signs of parenchymatous adipose and hydropic dystrophy; stroma was edematic with poor macrophagal and lymphocytic infiltration, vessels were plethoric; perivascular analysis revealed petechial hemorrhages. What type of myocarditis was developed in this case? A. Purulent B. Granulomatous C. Interstitial proliferative D. Serous focal E. Serous diffuse 9. Colonoscopy of a patient ill with dysentery revealed that mucous membrane of his large intestine is hyperemic, edematic, its surface was covered with greyandgreen coats. Name the morphological form of dysenteric collitis: A. Fibrinous B. Necrotic C. Purulent D. Ulcerous E. Catarrhal 10. A woman has been applying a new cosmetic preparation for a week that resulted in eyelid inflammation accompanyed by hyperemia, infiltration and painfulness. What type of allergic reaction was developed? A. I B. IV C. С III D. II E. V 11. A 22 y.o. woman has enlarged lymph nodes. Histologically: a lymph node contains lymphocytes, histiocytes, reticular cells, small and big Hodgkin's cells, multinucleated Sternberg cells, isolated foci of caseous necrosis. What disease are these changes typical for? A. Lymphosarcoma B. Lymphogranulomatosis C. Chronic leukosis D. Acute leukosis E. Lung cancer metastasis 12. A patient died under conditions of cardiovascular insufficiency. Autopsy results: postinfarction cardiosclerosis, myocardium hypertrophy and dilatation of its cavities, especially of its right ventricle. Liver is enlarged, its surface is smooth, incision revealed that it was plethoric, with darkred specks against the background of brownish tissue. Histologically: plethora of central parts of lobules; peritheral parts around portal tracts contain hepatocytes in a state of adipose degeneration. How are these liver changes called?
47
Welcome message from author
This document is posted to help you gain knowledge. Please leave a comment to let me know what you think about it! Share it to your friends and learn new things together.
Transcript
Page 1: Tests General 08

PATHOMORPHOLOGY GENERAL1. Histological examination of a 40 yo. man's thymus revealed decreased share of parenchymatous gland elements, increased share of adipose and loose connective tissue, its enrichment with thymus bodies. The organ's mass was unchanged. What phenomenon is it?A. Age involutionB. Accidental involutionC. HypotrophyD. Atrophy E. Dystrophy

4. A 9 m.o. child has delayed dentition, it is also out of order. Upper jaw configuration is horizontal ("high"palate); microscopically irregular mineralization of tooth enamel, wrinkled enamel prisms, some of them are vacuolized. Predentin zone is extended; there are solitary denticles. What disease is it?A. Early ricketsB. Hypervitaminosis DC. Late ricketsD. GoutE. Osteomalacia

6. A patient ill with diabetes mellitus felt acute pain in his right foot. Objectively: foot thumb is black, foot tissues are edematous, there are foci of epidermis desquamation, stinking discharges. What clinicopathological form of necrosis is it?A. InfarctionB. SequestrumC. Dry gangreneD. BedsoreE. Moist gangrene

8. Autopsy of a man who died from influenza revealed that his heart was slightly enlarged, pastous, myocardium was dull and had specks. Microscopical examination of myocardium revealed signs of parenchymatous adipose and hydropic dystrophy; stroma was edematic with poor macrophagal and lymphocytic infiltration, vessels were plethoric; perivascular analysis revealed petechial hemorrhages. What type of myocarditis was developed in this case?A. PurulentB. GranulomatousC. Interstitial proliferativeD. Serous focalE. Serous diffuse

9. Colonoscopy of a patient ill with dysentery revealed that mucous membrane of his large intestine is hyperemic, edematic, its surface was covered with greyandgreen coats. Name the morphological form of dysenteric collitis:A. FibrinousB. NecroticC. PurulentD. UlcerousE. Catarrhal

10. A woman has been applying a new cosmetic preparation for a week that resulted in eyelid inflammation accompanyed by hyperemia, infiltration and painfulness. What type of allergic reaction was developed?A. I B. IVC. С III

D. IIE. V

11. A 22 y.o. woman has enlarged lymph nodes. Histologically: a lymph node contains lymphocytes, histiocytes, reticular cells, small and big Hodgkin's cells, multinucleated Sternberg cells, isolated foci of caseous necrosis. What disease are these changes typical for?A. LymphosarcomaB. LymphogranulomatosisC. Chronic leukosisD. Acute leukosisE. Lung cancer metastasis

12. A patient died under conditions of cardiovascular insufficiency. Autopsy results: postinfarction cardiosclerosis, myocardium hypertrophy and dilatation of its cavities, especially of its right ventricle. Liver is enlarged, its surface is smooth, incision revealed that it was plethoric, with darkred specks against the background of brownish tissue. Histologically: plethora of central parts of lobules; peritheral parts around portal tracts contain hepatocytes in a state of adipose degeneration. How are these liver changes called?A. Pseudonutmeg liverB. AmyloidosisC. C. Nutmeg liverD. Liver cirrhosisE. Liver steatosis

13. In the surgical department of a hospital there was an outbreak of hospital infecti on that showed itself in often postoperative wound abscesses. Bacteriological examination of pus revealed aurococcus. What examination shall be conducted to find out the source of this causative agent among the department personnel?A. Microscopical examinationB. PhagotypingC. Estimation of antibiotic susceptibilityD. Biochemical identificationE. Serological identification

17. Autopsy of a man who died from chronic cardiovascular collapse revealed "tiger heart". Sidewards of endocardium a yellowishwhite banding can be seen; myocardium is dull, darkyellow. What process caused this pathology?A. Fatty vascularstromal degeneration B. Fatty parenchymatous degenerationC. Carbohydrate degenerationD. Hyaline degenerationE. Amyloidosis

18. A 48 y.o. patient was admitted to the hospital with complaints about weakness, irritability, sleep disturbance. Objectively: skin and scleras are yellow. In blood: conjugated bilirubin, cholalemia. Feces are acholic. Urine is of dark colour (bilirubin). What jaundice is it?A. Gilbert's syndromeB. ParenchymatousC. MechanicD. HemolyticE. CriglerNajjar syndrome

21. A 56 yo. patient has been suffering from thyreotoxicosis for a long time. What type of hypoxia can be developed?A. Tissue

Page 2: Tests General 08

B. MixedC. CirculatoryD. HemicE. Respiratory

22. Microscopical examination of a removed appendix revealed an edema, diffuse neutrophilic infiltration of appendix wall along with necrosis and defect of mucous membrane with affection of its muscle plate. What appendicitis form was developed?A. PhlegmonousB. ApostematousC. SuperficialD. UlcerophlegmonousE. Gangrenous

23. A patient is ill with diabetes mellitus that is accompanied by hyperglycemia of over 7,2 millimole/l on an empty stomach. The level of what blood plasma protein allows to estimate the glycemia rate retrospectively (48 weeks before examination)?A. Glycated hemoglobinB. CeruloplasminC. Creactive proteinD. FibrinogenE. Albumin

24. A 45 y.o. patient consulted a doctor about plaqueshaped formation on his neck. Histological examination of biopsy skin material revealed tumourous cells of round and oval form with thin ring of basophilic cytoplasma that resemble of cells of basal epidermal layer. What tumour is it?A. BasaliomaB. SyringoadenomaC. Epidermal cancerD. HydradenomaE. Trichoepithelioma

29. An experimental animal was first sensibilized whereupon an antigen dose was introduced subcutaneously. This injection resulted in the development of a fibrinous inflammation with alteration of vessel walls, basal substance and fibrous structures of connective tissue in form of mucoid and fibrinoid swelling and necrosis. What immunological reaction took place?A. Delayed type hypersensitivityB. Immediate hypersensitivityC. Reaction of transplantation immunityD. Normergic reactionE. Granulomatosis

30. Examination of cell culture got from a patient with lysosomal pathology revealed accumulation of great quantity of lipids in the lysosomes. What of the following diseases is this disturbance typical for?A. Wilson diseaseB. GalactosemiaC. TaySachs diseaseD. PhenylketonuriaE. Gout

31. A teenager was irradiated with high radiation dose that resulted in serious damages of lymphoid system, lysis of many lymphocytes. Restoration of normal hemogram is possible due to the functioning of the following gland:A. Adrenal

B. PancreasC. ThyroidD. ThymusE. Liver

32. Autopsy of a 58 y.o. man revealed that bicuspid valve was deformed, thickened and unclosed. Microscopically: foci of collagen fibrilla are eosinophilic, react positively to fibrin. The most probably it is:A. Fibrinous inflammationB. HyalinosisC. AmyloidosisD. Fibrinoid swellingE. Mucoid swelling

34. A patient with clinical presentations of immunodeficiency went through immunological examinations. They revealed significant loss of cells that form rosettes with erythrocytes of a ram. What conclusion can be made according to the analysis data?A. Decrease of natural killer cell rateB. Insufficiency of effector cells of humoral immunityC. Decrease of T-lymphocytes rateD. Decrease of complement system rateE. Decrease of B-lymphocytes rate

35. Autopsy of a patient who suffered from croupous pneumonia and died from pneumococcal sepsis revealed 900 ml of turbid greenishyellow liquid in the right pleural cavity. Pleural leaves are dull, plephoric. Name the clinicopathological form of inflammation in the pleural cavityA. PhlegmonB. Chronic abscessC. EmpyemaD. Fibrinous inflammationE. Acute abscess

36. Mucous membrane of the right palatine tonsil has a painless ulcer with smooth lacquer fundus and regular cartilaginous edges. Microscopically: inflammatory infiltration that consists of lymphocytes, plasmocytes, a small number of neutrophils and epithelioid cells; endovasculitis and perivasculitis. What disease is it?A. Pharyngeal diphtheriaB. Ulcerous necrotic Vincent's anginaC. ActinomycosisD. SyphilisE. Tuberculosis

37. A 27yearsold woman has been suffered by diabetes mellitus since she had been a child. During the last years the arterial hypertension and proteinuria have occurred. She has died with signs of uremia. What typical changes were found out in autopsy?A. “Cor villosum”B. “Tiger heart”C. “Armor heart”D. Mesenchymal fatty dystrophy of the heartE. Purulent pericarditis

38. A 43yearsold patient has a burn of right hand. The exfoliation of epidermis and formation of bubbles filled by semitransparent fluid is manifested in the palm and the back surface of the hand. What kind of inflammation occurs in that case?A. A.Serous

Page 3: Tests General 08

B. B.PurulentC. C.FibrinousD. D.PutrefactiveE. E.Catarrhal

39. A patient was admitted in the hospital with diagnosis of trichinellosis. What kind of inflammation develops within the muscles around the parasites?A. Proliferative granulematousB. Exudative fibrinousC. Exudative purulentD. Exudative serousE. Proliferative interstitial

40. A 22yearsold patient was admitted in hospital with complaints on heavy nasal breathing. During the examination of nasal cavity it was found the thickened mucous membrane, a lot of mucus and nodular infiltrates without erosions. It was diagnosed as rhinoscleroma of nose. It was took the biopsy. What typical morphological changes may be found? A. Granulomas with Mikulicz’s cellsB. Granulomas with Virchow’s cellsC. Granulomas with Langhan’s cellsD. Granulomas with foreign body cellsE. Interstitial inflammation

41. During the histological examination of the fragment of mucous membrane of urine bladder of the patient suffered with chronic cystitis it was found out: the transitional epithelium focally is replaced by stratified nonkeratinized epithelium. These changes can manifest development of:A. MetaplasiaB. DysplasiaC. Leukoplakia D. HyperkeratosisE. Hyperplasia

42. A 1/3 of the liver with a multicellular echinococcus was resected during the operation. What typical changes of the liver tissue surrounding the parasites were found out by the pathologist during the histological examination of the resected part of the organ?A. Formation of nonspecific granulomasB. Flabby consistence of the liverC. Formation of specific granulomasD. Exudative tissue reactionE. Hydropic dystrophy

43. During the autopsy it was found the enlarged firm liver with rounded margins; the surface of the cut looks nutmeglike of yellowbrown colour with darkred spots. What pathologic process do underlie of those changes?A. Chronic venous hyperemiaB. Arterial hyperemiaC. Arterial ischemiaD. Fat embolismE. Chronic bleeding

44. There is a redness of skin above the site of obturation, if the thrombus obstructed the main artery of upper extremity. Call the kind of arterial hyperemia appearing as a result of disturbance of bloodstream.A. CollateralB. AngioneuroticC. InflammatoryD. Physiological

E. After anemia

45. A 23yearsold patient got a lesion of the liver because of trauma. In time a structure and functions of that organ was restored completely. What kind of regeneration did occur in that case?A. RestitutionB. Pathologic regenerationC. Physiologic regenerationD. MetaplasiaE. Substitution

46. A 4yearsold girl was operated because of an acute appendicitis. During the operation in the retroperitoneal space it was found that the right kidney is less by 1/3 in comparison with the left one. The diameter of the right renal artery was 0,3 cm, the left one was 0,4 cm. What pathologic process did occur in that case?A. HypoplasiaB. Pathologic atrophyC. Physiologic atrophyD. AplasiaE. Agenesia

47. A patient has suffered with mitral insufficiency that complicated by chronic cardiac insufficiency with grossly marked edema of the lower extremities. Suddenly cyanosis of the face is manifested with attempt standing up and he has died. What the most probably complication developed in patient?A. Thromboembolism of the pulmonary arteryB. Venousarterial reactionC. Infarction of myocardiumD. Chronic venous hyperemiaE. Hypertension stroke

48. A child fell down the tree and got a simple fracture of the one of hand’s bones. In a time healing occurred. Call the kind of regeneration and cells, which taking part in restoration.A. Reparative regeneration, osteoclasts and osteoblastsB. Physiological regeneration, octeoclastsC. Pathological regeneration, osteoclastsD. Intracellular regeneration, osteocytesE. Cellular regeneration, osteoclasts and osteoclasts

49. A pilot has died because of depressurization of the cabin of the airplane. In autopsy it was found out a gross marked rigor mortis, emphysema of subcutaneous tissue of the body and face. There were also edema and perivascular hemorrhages within the lungs. There was a foamy and fluid blood within the veins. The histological examination was showed a lot of bubbles within the vessels of all inner organs, a fat dystrophy of the liver and a quantity of small gray softening areas in the brain. Call the more probable cause of those changes.A. Gas embolismB. Air embolismC. Tissue embolismD. ThromboembolismE. Fat embolism

50. A man has died in 8th day since beginning of the disease. It was diagnosed dysentery. During the autopsy it was found out a thickened wall of the sigma and rectum, fibrinous membrane on the surface of mucous membrane.

Page 4: Tests General 08

Histologically: there is a deep necrosis of mucous membrane with infiltration of necrotic masses with fibrin. What kind of colitis does correspond to those changes?A. DiphtheriticB. CatarrhalC. UlcerativeD. ChronicE. Gangrenous

51. A patient suffered with rheumatic fever with a mitral valve disease of the heart for a long time. He has died as a result of cardiacpulmonary insufficiency. During the autopsy it was found out a brown induration of the lungs. Call kind of hemodynamic disorders causing those changes.A. Chronic left ventricle failureB. Chronic right ventricle failureC. Acute left ventricle failureD. Acute right ventricle failureE. Portal hypertension

52. A patient suffered by stenosis of mitral valve of 4th stage has died as a result of cardiacpulmonary insufficiency. Autopsy was showed a hemosiderosis of the lungs. Call the kind of hemodynamic disorder in the lungs.A. Local venous hyperemiaB. General venous hyperemiaC. Arterial hyperemiaD. Inflammatory hyperemiaE. Hemorrhage

53. A patient has died as a result of cardiac insufficiency. In anamnesis he had got a pulmonectomy in account of cyst of the right lung. During the autopsy enlarged left one was found out. Call the pathologic process in the left lung.A. Replacement hypertrophyB. Neurohumoral hypertrophyC. Dyscirculatory atrophyD. Dysfunctional atrophyE. Physiologic hypertrophy

54. A patient has undergone to amputation of lower extremity. In a time a painful nodules appeared in a stump. Amputatious neuromas were found out during the microscopical examination. What kind of pathological processes does those formations relate to?A. RegenerationB. DystrophyC. InflammationD. HyperemiaE. Metaplasia

55. After deep burns of the skin a patient has got a keloid scarring. What kind of pathologic processes does those formations relate to?A. Incomplete regeneration (substitution)B. Complete regeneration (restitution)C. AtrophyD. HypertrophyE. Metaplasia

56. A patient suffered with urolithiasis has died as a result uremia. Autopsy is showed an enlargement of right kidney, its parenchyma has been thinned; the pelvis and calices have been extended, filled with fluid. In the orifice of ureter there has been a calculus. Call the pathologic process in the right kidney.A. Hydronephrosis

B. Replacement hypertrophyC. HyperplasiaD. Neoplasm of kidneyE. Pyelonephritis

57. A woman suffered with dysfunctional metrorrhagia was made a diagnostic abortion. Histologically in the scrape there were a lot of small stamped glandulas covered by multirowed epithelium. The lumens of some glandulas were cystically extended. Call the variant of general pathologic process in the endometrium.A. Glandularcystic hyperplasia of endometriumB. Atrophy of endometriumC. Metaplasia of endometriumD. Neoplasm of endometriumE. Hypertrophic growth

58. A 13yearsold child was admitted into the hospital with complaints of fever, weakness, and intense pain in the right thigh. From anamnesis it was defined that he has bitten by dog 3 days ago. During the visual examination it was found out redness, swelling and acute painfulness along right thigh. After excision it was made the histological examination, which was showed a considerable accumulations of polymorphonucleus leukocytes between the mussel fibers partly undergone by purulent fusion. Diagnose that condition.A. PhlegmonB. AbscessC. GangreneD. EmpyemaE. Croupous inflammation

59. A patient with severe fracture of both femoral bones has died in a 4 hours after trauma. What additional staining of microscopic sections is it necessary to make for exacter diagnosis of the cause of death? A. The lungs and brain by Sudan IIIB. The lungs and kidney by CongoredC. The lungs and liver by Perl’s reactionD. The brain and kidney by CongoredE. The kidney and heart by toluidinblue

61. A 59yearsold woman was admitted into the hospital with signs of exicosis being in deep shock. After her death it was made autopsy showed presence of vibrio cholerae, moderate mononuclear infiltrations in the mucosal lamina propria of intestine. What kind of shock was the cause of death of the patient?A. HypovolemiaB. CardiogenicC. AnaphylacticD. TraumaticE. Toxic

62. A cattle worker has died with signs of severe intoxication in a 2nd day after beginning of disease. Autopsy was showed the enlarged flabby spleen, on the surface of the cut of darkcherry coloured; the scrape of the pulp is abundant. The leptomeninges are swelling, infiltrating by blood of darkred colour (“a red cap”). It was defined a microorganism – bacillus anthracis – and diagnosed anthrax. What kind of inflammation does occur in that disease?A. HemorrhagicB. PhlegmonousC. Fibrinous

Page 5: Tests General 08

D. PurulentE. Putrefactive

63. A 65yearsold patient suffered by thrombophlebitis of the deep veins of both legs has died suddenly. Autopsy was showed free lying dry friable red masses with a dull crimped surface within the truncus pulmonalis and bifurcation of the lung artery. What process within the vessels did pathologist find?A. ThromboembolismB. ThrombosisC. Tissue embolismD. Postmortem clotE. Hemangioma

64. A patient was admitted into the hospital with complaints to feel sick, pains in the epigastria after meal. During the gastroscopy it was found out hyperemia, edema of mucous membrane of the stomach, an excessive amount of viscous grayish mucus. What kind of inflammation did develop in the stomach?A. CatarrhalB. FibrinousC. SerousD. HemorrhagicE. Purulent

65. During the autopsy it was found out: the lungs are firm, have brown colour because of accumulation of endogenous pigment. It’s known that the patient has suffered with venous congestion in the pulmonary circulation. What process did cause such changes?A. HemosiderosisB. CalcinosisC. JaundiceD. MelanosisE. Porphiria

66. During the autopsy it was found out the cavity 2,5x1,5 sm within the right frontal part of the brain. It was filled with transparent fluid and its walls were smooth and of brownish colour. What process did develop in the brain?A. Cyst as an outcome of hemorrhage.B. Grey softening of brain.C. Abscess of brain.D. Birth defect of brain.E. Cyst as a outcome of gray softening

67. During the histological examination of the fragment of cervix it was found out the area of inflammatory infiltration with involvement of walls of small vessels. The infiltrate was composed of plasma cells, lymphocytes, epithelioid cells and areas of sclerosis and hyalinosis. What disease does such picture characterize?A. Syphilis.B. Tuberculosis.C. Leukoplakia.D. Erosion of cervix.E. Condyloma.

68 During the histological examination of the biopsy it was found out the granulomas composing of lymphocytes, epithelioid cells, plasma cells, macrophages with foamy pale cytoplasm (Mikulicz’s cells), a lot of hyaline balls. What disease do you think about?A. Rhinoscleroma.B. Leprosy.

C. Syphilis.D. Tuberculosis.E. Actinomycosis.

69. The patient with severe intoxication has died. During the autopsy it was found out: the left tonsil was enlarged, firm; in section it contained a lot of cavities filled by yellow–greenish masses. The tissues of the neck, and mediastinum were undergoing with suppurative fusion. Microscopically: there were a lot of abscesses, in the center of which intensive basophilic formations were defined. Those formations consist of short sticklike elements, surrounded with radiating projections (“ray fungus”). Such changes characterized: A. Actinomycosis.B. Amebiasis.C. Brucellosis.D. Tuberculosis.E. Scarlet fever.

70. A 34yearsold woman has got pain, redness and swelling of finger on account of careless use of iron. The blister filled by transparent yellowish fluid appears in some minutes. What pathologic process do such signs characterize?A. Exudative inflammation.B. Traumatic edema.C. Hypertrophy.D. Proliferative inflammation.E. Hydropic dystrophy.

71. The patient with chronic glomerulonephritis has died with accompanied signs of uremia. The cadaver had specific uremic smell. Macroscopically: there were a lot of pilelike whitishgrayish puttings on the surface of pericardium. The vessels filled with blood became visible after removal of such formations. What process did develop in the pericardium?A. Fibrinous inflammation.B. Organization.C. Proliferative inflammation.D. Hemorrhagic inflammation.E. Arterial hyperemia.

72. The patient complains of local pain in the back of the head, increasing of the temperature in that area. Macroscopically: there is a conical infiltrate of purpleblue colour with yellowgreenish apex, which protrudes under the surface of the skin. Diagnose it:A. Furuncle.B. Phlegmona.C. Abscess.D. Fistula.E. Empyema.

73. During the histological examination of the biopsy the pathologist has found out granulomas within the livers. They contain mainly Tlymphocytes and epithelioid cells, and solitary giant Langhan’s cells. In the center of granulomas there was a small area of caseous necrosis. What pathologic process is characterized by those changes?A. Proliferative inflammation.B. Coagulative necrosis.C. Liquefactive necrosis.D. Neoplasm.E. Exudative inflammation.

Page 6: Tests General 08

74. A 5yearsold girl suffered from diphtheria. She has died in three days because of asphyxia caused by diphtheria’s croup. During the autopsy it was found out, that mucous membranes of larynx, trachea and bronchi were thickening, swelling, dull, covering by grayish membranes, which easily came off. What process did develop in the larynx?A. Croupous inflammation.B. Serous inflammationC. Diphteritic inflammationD. Proliferative inflammationE. Suppurative inflammation

75. As a result of a microscopic examination of the fragment of the skin the granulomas were found out; they composed of a lot of macrophages with a few amount of lymphocytes and plasma cells. Besides large macrophages with fat vacuoles occurred, the microorganisms were packed in those vacuoles as “cigars in the box” (Virchow’s cells). Granulation tissue had a good vascularization. What disease are such granulomas characterized for?A. LeprosyB. Tuberculosis.C. Syphilis.D. Rhinoscleroma.E. Actinomycosis.

76. During the microscopic examination of bioptic fragment of the skin the granulomas were found out containing epithelioid cells, surrounded with Tlymphocytes. Between the epithelioid cells the solitary giant polynuclear Langhan’s cells located. There were areas of caseous necrosis in the center of some granulomas. Blood vessels were absent. What disease do such changes characterize?A. Tuberculosis.B. Syphilis.C. Leprosy.D. Rhinoscleroma.E. Hodgkin’s disease.

77. In a biopsy of a сervix uteri of a26yearold women the diagnosis following was established: pseudoerosion. What microscopical changes has the pathologist revealed? A. Local changes of a stratified squamous epithelium

on single-layer prismatic one B. Cellatypia of an epithelium of an mucosal epithelium C. Keratinization of an epithelium of an mucosal

epithelium D. “Carcinomatous pearls” E. Local inflammation and necrosis in mucosa

80. Inheritable predilection to development of lung emphysema is connected with genetic disturbance, which one shows in decreasing production of:A. Alpha1antitrypsin B. Creactive proteinC. Antistreptolysin 0D. Hyaluronidasa E. Complement

82. Gangliosidlipidosis is characterized by deficiency of enzyme alpha –hexoaminidaze and accumulation of gangliosides in lysosomas of nervous cells.It is connected with genetical defect, which is determined as: A. Defect of one geneB. Translocation of a gene C. Defects of several genes D. Trisomy of 21 chromosoma

E. Defect of telomere Хchromosoma

83. During the autopsy the pathologist has marked: the expressed linear shrinkage and thickening of extremities with formation on a skin of large tucks. The head is increased, nose is saddle, the oral cavity is semiopen, tongue is thick, neck is short, bodies of spondyles are dwarfed, hypoplasia of thorax is combined with hypoplasia of lungs. Which inherent malformation of a musculoskeletal system is it characteristic for?A. Lethal micromelia B. Chondroplasia C. Imperfect bone formation D. Inherent marble bones E. Inherent Oppenheim myatonia

84. In the autopsy of 48yearold man [miner which got hematite] the pathologist has found out increased brownred dense lungs. Microscopic picture is: a moderate pneumosclerosis, submiliary and miliary nodules, which consist of dust cells with spots (positive reaction to iron). In lymph nodes there are a lot of dust and considerable diffuse sclerosis. Which professional disease is it? A. Red siderosis B. Black siderosis C. AluminosisD. BerilliosisE. Anthracosis

85. In the autopsy of a body of the miner, which got hematite, the pathologist has found out: black lungs, similar on lungs at an anthracosis. Microscopically: moderate pneumosclerosis, submiliar and miliar nodules, which consist of dust cells with spots [positive reacton to iron]. In lymph nodes there are a lot of dust and considerable diffuse sclerosis. Your diagnosis? A. Black siderosis B. Red siderosis C. Aluminosis D. Berilliosis E. Caplan syndrome

90. A 45yearold man has died because of pulmonarycardiac insufficiency. In autopsy the pathologist has found out croupous pneumonia in lower lobe of left lung, 350ml of greenishyellow fluid in the left pleural cavity. Microscopically it contained many neutrophils. Call the complication of pneumoniaA. Empyema of pleuraB. Fibrinous pleuritisC. PneumothoraxD. Hydrothorax E. Hemothorax

98. A 60yearold miner has died because of chronic pulmonarycardiac insufficiency. In autopsy the pathologist has found out: areas of dystelectasis, induration, pneumosclerosis in the both lungs, local emphysema in the apex. Cut surface was slatyblack. Diagnose this disease.A. Anthracosis B. Silicosis C. Talcosis. D. AsbestosisE. Aluminosis.

108. In 56 year –old patient has suffered from bronchoectatic disease and hemoptysis, the edema of face

Page 7: Tests General 08

and waist have appeared. The protein (33 mg/l) was found in urine. Pulmonary hemorrhage was the cause of patient’s death. In autopsy: enlargement of kidneys was found; the kidneys were densed with lardaceus surface of section. Histologically: the deposition of homogenous eosinophilic masses colored with Congo red and given of metachromasia with methyl violet color in glomeruli and canals were found. What pathological process took place in the patient?A. AmyloidosisB. Grainish degeneration;C. Fatty degeneration;D. Mucoid degeneration;E. ;Hyalinosis;

111. In 45year old patient died from sudden cardiac death the symmetrical type of adipose heart of third degree; the rupture of right ventricle’s wall with hemopericardium and redundant accumulation of fat under epicardium were found in autopsy. Microscopically: the adipose tissue grows from epicardium into myocardium with atrophy of fibers of muscle. What process is more probable?A. Adipose heart;B. Hypertensive disease;C. Ischemic heart disease;D. Fatty degeneration of myocardium;E. Acute myocardial infarction;

112. The 48yearold patient, suffered from fibrouscavernous tuberculosis, has complained of weakness, reduction of daily secretion of urine, edema of the body and extremities and increasing of blood pressure to 180/90. The increasing of protein, presence of hyaline and grain cylinders and erythrocytes were found under analysis of urine. The patient has died in a month because of the insufficiency of kidney. In autopsy, the enlargement of the heart and "lardaceus" kidneys with mass of more than 240 g were found. What is the complication of fibrosecavernouse tuberculosis?A. Аmyloidosis;B. Glomerulonephritis;C. Nephrotic syndrome;D. Pyelonephritis;E. Nephrosclerosis;

114. In patient with jaundice the following data were established: in serum the increasing of bilirubin because of the unconjugated form; in faeces and urine increasing of stercobilin; the level of conjugated (direct) bilirubin in serum is normal. What type of jaundice takes place?A. Haemolytic jaundice;B. Jaundice of newborns;C. Parenchymatous (hepatic) jaundice;D. Gilbert’s disease;E. Mechanical (posthepatic) jaundice;

115. In autopsy of the 83yearold patient’s body, has died because of stomach cancer, the heart and the liver were diminished in size, condensed, with brown color. What pathological process took place in these organs?A. Sclerosis;B. Mesemchymal degeneration;C. Parenchymal degeneration;D. Necrosis;E. Brown atrophy

116. Fragment of dead tissue, which can’t be autolized, replaced by connective tissue and which is localized among alive tissue is named…

A. SequestrumB. Dry gangreneC. Wet gangreneD. InfarctionE. Caseous necrosis

117. The 46year old woman has suffered from rheumatic fever with combined mitral heart disease. In autopsy the leaflets of mitral valve are thicken, intertwisted and have stony dense. What pathological process has led to these changes?A. Amyloidosis;B. Hyaline changes;C. Dystrophyc calcification;D. Metabolic calcification;E. Metastatic calcification;

118. Patient is suffered from cholera, which is during with dehydratating, cyanosis and convulsions. In result of massive infusion therapy the exicosis is diminished, but anuria is remained. Patient is dead of uremia. Choose correct position for this case.A. Necrotic nephrosis with cortical necrosis takes place in the kidneys,B. Choleral typhoid is developed,C. Development of uremia is connected with acute glomerulonephritis,D. Fibrinouse colitis is found in autopsy,E. Exicosis is due to action of virus exotoxin.

119. Please, choose incorrect statement.A. Degeneration is characterized by disorder of tissue nutrition and in every time lead to distraction of nucleus and whole cellB. There are extracellular and intracellular accumulationsC. Pathological calcification may be in two forms: dystrophic and metastatic calcificationD. Degenerations may be caused by hereditary factorsE. Degenerations can appear in one organ or in the whole body

120. In autopsy: enlargement of kidneys was found; the kidneys were densed with lardaceus surface of section. Histologically: the deposition of homogenous eosinophilic masses colored with Congo red and given of metachromasia with methyl violet color in glomeruli and canals were found. What pathological process took place in the patient?A. AmyloidosisB. Grainish degeneration;C. Fatty degeneration;D. Mucoid degeneration;E. ;Hyalinosis;

121. Choose one incorrect statement:A. The mechanism of decomposition is a base of genetic storage diseases;B. Decomposition (phanerosis) – disintegration of membranous structures of cells and intercellular matrix;C. Infiltration – redundant accumulation (deposition) of metabolites into the cells and intercellular matrix; D. Perverted synthesis – synthesis of abnormal substances in the cells and tissues;E. Transformation – formation of one type of metabolism products from common initial substances for protein, fats and carbohydrates.

122. Intracellular accumulation of fat in liver is

Page 8: Tests General 08

characterized by every statements, except one:A. Fat is accumulated in extracellular spacesB. There are small fat vacuoles in cytoplasm around the nuclei of hepatocytesC. Liver enlarged and becomes yellowD. Its consistency is soft and greasyE. Fat is stained by special stain Sudan 3, and becomes orangered color

123. A 55 year old man has died after chronic glomerulonephritis, chronic renal insufficiency. In the autopsy the pathologist has found out characteristic changes in kidneys for this disease, also fibrinous pericarditis, pleuritis, bronchitis. Call the cause of the fibrinous inflammation in serosal and mucosal layers. A. Uremia.B. Hypolipidemia C. HyperlipidemiaD. Arterial hypertension E. Arterial plethora

140. During autopsy of the patient died from chronic cardiac insufficiency the enlarged dense rusty colored lungs with growth of gray color connective tissue around bronchi and vessels were found. How can we call this process in lungs?А Brown induration of the lungВ Hemorrhagic pneumoniaС Interstitial emphysemaD Chronic obstructive emphysemaE Primary idiopathic emphysema;

143. In 77yearold patient suffered with atherosclerosis the pain has appeared in the right foot. The foot is enlarged in size, its skin has black color and is macerated; the demarcation line is not clear. What pathological process takes place in a patient?A. Wet gangreneB. Coagulative necrosisC. SequestrumD. Dry gangreneE. Noma

145. A 65 year old patient, who suffered from arteriosclerosis, has been hospitalized in surgical department because he had purulent peritonitis. Thrombosis of mesenteric arteries was found during operation. What is the most probable cause of peritonitis?A. Hemorrhagic infarctionB. Angiospastic ischemiaC. Angioneurotic edemaD. StasisE. Chronic congestion

148. Under microscopic investigation the postinfarction cardiosclerosis has been found out. Around cardiosclerotic area myocardiocytes were enlarged in size and had large hyperchromic nuclei riched in DNA. What process taking place in myocardiocytes is more probable?A. Regenerative hypertrophy B. Physiologic regenerationC. Complete reparative regenerationD. Pathologic regenerationE. Hypertrophy because of increased workload

149. 56yearold patient has suffered from rightside lowerlobar pneumonia with expectoration of mucus with

pus. In autopsy in 910 segments of the right lung the cavity with dense walls filled with purulent masses, was found. The whitish path comes from the cavity toward the radix of the lung. Microscopically in was established that the cavity is divided from saved lung tissue with thin membrane, which consists of two layers: internalgranulation tissue, and external –connective tissue. What diagnosis is more probable?A. Chronic abscess;B. Pulmonary gangrene;C. Acute pulmonary abscess;D. Chronic pneumonia;E. Bronchoectatic disease

150. Cyanosis, enlargement of the liver, edema of the low extremities as a result of insufficiency of the right ventricle of the heart were found in a patient. What is the cause of the development of such insufficiency?A. Hypertension of the small circle of the blood circulation B. Functional shunts in lungsC. Increasing of the venous pressureD. HypercathecholemiaE. Cardiogenic cirrhosis of the liver

153. In 53 yearold patient, who has suffered by bronchoectatic disease and hemoptysis, the edema of face and waist have appeared. The protein (33 mg/l) was found in urine. Pulmonary hemorrhage was the cause of patient’s death. In autopsy: enlargement of kidneys was found; the kidneys were dense with lardaceous surface of section. Histologically: the deposition of homogenous eosinophilic masses colored with Congo red and given of metachromasia with methylviolet color in glomeruli and canals were found. What pathological process took place in the patient?A. Amyloidosis;B. Fatty degeneration;C. Mucoid degeneration;D. Grainish degenerationE. Hyalinosis

155. In the case of rheumatic heart disease, pericarditis is characterized by:A. “Shaggy heart” (“cor villosum”)B. “Cor pulmonale”C. “Tiger heart”D. Fatty changeE. “Bull heart”

156. In a 45yearold patient the ulcerativenecrotic damage of the mucosa of the oral cavity takes place; also the spread lymphadenopathy, slight spleno and hepatomegaly, diffuse hemorrhages in the skin and mucous membranes were found out. In blood analysis the increasing of leukocytes (to 100 10 per 1 ml) at the account of lymphoblasts takes place. What diagnosis is more probable?A. Acute lymphoblastic leukemia B. Chronic myelocytic leukemiaC. Chronic lymphocytic leukemiaD. PlasmocytosisE. Acute promyelocytic leukemia;

159. Chronic venous congestion of organs, hypertrophy of the left heart ventricle with cardiosclerosis, volumetrical white-yellowish plaques in intima of aorta with atheromatous masses in the center, which are deeper in the wall, are found in the result of autopsy of 70year man, who

Page 9: Tests General 08

died of heart insufficiency and who suffered from angina pectoris, hypercholesterinemia and obesity. Which pathological process is more possible in aorta?A. Atheromatosis B. LipoidosisC. ArteriolosclerosisD. LiposclerosisE. Calcification

161. All statements are correct in the relations of hyalinosis, except one:A. The hemorrhage and hemoliysis play important role in the hyalinosis formationB. Hyaline is accumulated in the connective tissue and looks like cartilageC. Mainly arterioles and arteries undergo to hyalinosisD. Hyalinosis is rare reversible processE. There are not macroscopical changes of the organs usually, but hyalinosis may lead to deformation and wrinkle of the organs

166. In 60yearold patient, during examination the cancer of a prostate gland with metastases in the lower parts of the column, pelvic bones and pathological fracture of a femur was found out. What pathological process is a basis of the development of metastases? A. Tissue embolism B. Bacterial embolism C. Airembolism D. Embolism by foreign matters E. Necrosis of tissue

167. In patient with chronic cystitis during the investigation of biopsy of a mucosa of urinary bladder the foci of a squamous nonkeratinized epithelium are found out. It can be a sign of development of:A. MetaplasiaB. LeukoplakiaC. HyperplasiaD. Dysplasia E. Hyperkeratosis

168. During the laparotomy in 49 yearold male patient, the tumor has been found out in the field of a sigma with growth through all its layers and an occlusion of the lumen of an intestine. The biopsy has been taken and colonostoma has been overlapped. The clinical diagnosis after operation: a cancer of sigma. What kind of tumor is growth in relation to tissues? A. InfiltrativeB. Expansive C. Endophytic D. Exophytic E. Multicentric

169. In 36 yearold patient a dark brown color patch on skin of a dextral foot has been resected. The histologic research has not been carried out. In 5 months in dextral inguinal area a conglomeration of lymph nodes has appeared. At histologic research of one of them the metastasis of melanoma has been found out. What pathway of spreading of tumor takes place?A. Limphogenous spreadB. Hematogenous spreadC. AnaplasticD. MixedE. Perineural

170. At the autopsy of 60 yearold male patient, in the myocardium of the front wall of the left ventricle of the heart a gray dense focus of 5 х 4 cm, of irregular shape with clear borders, fibrous structure has been found out. What pathological process in the myocardium has revealed the pathologist? A. Postinfarctional cardiosclerosis B. Diffuse smallfocal myocardiosclerosis C. Myocarditis D. Petrification of the myocardiumE. Infarction

171. In 40yearold patient, the tumor, which grew under skin of spine was resected. The histologic diagnosis: a lipoma. What principle of the tumors’ classification did the pathologist use when created his conclusion? A. Gistogenesis B. Of biochemical features C. Of ultrastructural features D. Of physicochemical features E. Macrostructure of an organ

172. At histologic research of a biopsy specimen from an auricle of a heart of a patient with rheumatic disease the foci of a mucoid swelling, fibrinoid necrosis of a connective tissue has been found out. What immune response has developed in tissues of the auricle of the heart? A. Hypersensitivity of an immediate typeB. Hypersensitivity of a delayed typeC. Reaction of the transplantative immunityD. Normergic reaction E. Exudative reaction

178. Call a kind of an infarct according to macroscopic signs, which is characteristic in myocardium. A. White with a hemorrhagic halo B. Hemorrhagic C. Anemic D. Mixed E. Red

179. What type of exudate appears in pericardium cavity at a rheumatic pericarditis? A. Serous-fibrinous B. Putrefactive C. HemorrhagicD. Purulent E. Catarrhal

180. At autopsy of a patient died because of a cerebral hemorrhage, strongly enlarged dense and anemic kidneys ( size :6х3х2см weight 60.0,) with a uniformly smallgranulated surface and with uniformly thinned cortex on a cutsurface have been found out. The changes in kidney are a sign of: A. Of arteriosclerotic nephrosclerosis B. Of atherosclerotic nephrosclerosis C. Secondary shrinkage of kidney D. Gouty (podagric) kidneys E. Amiloidshrinkaged kidneys (“lardaceous kidneys”)

183. A 50yearold man had rheumatic mitral disease. He has arrived in clinic with sings of active rheumatism. On a background of increasing cardiovascular insufficiency has developed lefthand hemyplegia. The patient has died. What changes of a brain have caused development lefthand

Page 10: Tests General 08

hemyplegia?A. Infarction of a brainB. Hematoma.C. Atherosclerosis of brain’s vessels. D. Edema of a brain.E. Meningitis.

187. At histologic examination of tuberculoma resected from right lung a necrosis in center is found out. Call a kind of the necrosis.A. CaseousB. Fatty C. LiquefactiveD. CoagulativeE. Gangrene

194. A woman, which was in a bed in forced position for a long time, has died because of a tumoral dissemination (multiple metastases and carcinomatosis of a peritoneum) of mucinous cystadenocarcinoma. During the autopsy the large areas of necrosis of the skin and underlying soft tissues of sacral part were found out. Call a kind of a necrosis. A. BedsoreB. Infarct C. Sequester D. Caseous necrosisE. Waxylike (Zenker’s) necrosis

198. The examination of the child with measles showed the nonclear border edematous fluctuated areas of redblack color in the soft tissues of the cheeks and perineum. What complication did develop in the child? A. Wet gangrene (noma)B. Dry gangreneC. Gas gangreneD. BedsoreE. Trophic ulcer

202. The presence of the grains of chromatin in a focus of caseous necrosis is a manifestation of: A. KaryorhexisB. KaryolysisC. KaryopyknosisD. Mitotic activity of nucleiE. Apoptosis

205. In 77yearold patient suffered with atherosclerosis the pain in the right foot has appeared. The foot is enlarged in size, its skin has black color and is macerated; the demarcation line is not clear. What pathological process takes place in the patient?A. Wet gangrene B. Coagulative necrosisC. SequestrumD. Dry gangreneE. Noma

206. The 30yearold man has had for two months lacrination, pruritic palpebras, rhinitis with mucus. All symptoms disappeared after treatment by desensebilizators. What type of hypersensitivity occurred in patient?A. The type IB. The type IIC. The type IIID. The type IVE. The type V

207. The 35yearold female was admitted into the hospital with nephrolithiasis. Anaphylactic shock was appeared after intravenous injection of contrast medium. Which cells take part in development of this reaction?A. Tissue’s basophilsB. FibroblastsC. EpitheliocytesD. MyocytesE. Neutrophils

208. Histologic investigation of thyroid gland has showed destruction and atrophy of follicles, diffuse lymphoid infiltration with formation of lymphoid follicles in the stroma. Call the group of diseases with respect to this thyroiditis.A. Autoimmune specificB. Autoimmune nonspecificC. BacterialD. InfectiousallergicE. Viruses infectious

209. Ultrastructural investigation of tissue’s biopsy has showed considerable increase of lyzosomes in the cytoplasm of macrophages within the inflammative infiltrate. What is connected with appearance of this morphologic process?A. Activation of phagocytosisB. Proliferation of cellsC. Torpid phagocytosisD. Activation of apoptosisE. Torpid apoptosis

210. In biopsy of stomach in patient with autoimmune gastritis it was found out: infiltration by lymphocytes and macrophages in mucous layer. Which type of hypersensitivity is connected with these morphologic changes?A. The type IVB. The type II C. The type VD. The type IE. The type III

211. In biopsy of lymph node it was found out a lot of lymphoid follicles with large centers of duplication, increasing of mitoses. Which process is characterized by these morphologic changes?A. Antigenic stimulation with follicular hyperplasiaB. Atrophy of lymphoid tissueC. LymphosarcomaD. Hodgkin’s diseaseE. Metastases of cancer

212. It was found stopping of breathing and blood circulation in patient after traffic accident. Resuscitation was successful. Call the that condition of patient.A. Clinical deathB. Social deathC. Biological deathD. Physiologic deathE. Agony

213. Call the cells of the immune system that secrete immunoglobulins:A. Plasma cellsB. Macrophages

Page 11: Tests General 08

C. TlymphocytesD. Dendritic cellsE. Natural killer cells

214. Which cells are injured in patient with HIVinfection?A. Tcells-helpersB. BlymphocytesC. Natural killer cellsD. MacrophagesE. Eosinophils

215. The 35yearold man has received prolonged immunosupressive therapy after transplantation of kidney. He has died because of generalized cytomegaly. Call the kind of immunodeficiency in patient.A. MedicamentousB. XradiatingC. BacterialD. PrimaryE. Ecological

216. The 30yearold patient with transplanted kidney has received prolonged immunosupressive therapy and he has died because of intoxication. Microscopic examination showed giant cells with large nuclei encircled by ringslike brightening, which looked as “owleye”, located in the kidneys, liver, pancreas, lungs. Call this disease.A. Cytomegalovirus infectionB. TuberculosisC. SyphilisD. LeprosyE. Bubonic plaque

217. In biopsy of the lymph node it was found out granuloma with epithelioid and giant cells. Call the origin of epithelioid cell’s development.A. MonocytesB. PericytesC. NeutrophilsD. LymphocytesE. Epithelium cells

222. In biopsy of transplantanted kidney it was found out: diffuse infiltration of stroma by lymphocytes, plasma cells, lymphoblasts, plasmablasts, necrotic arteriitis. Which pathologic process was appeared in organ?A. Immune mutilationB. Acute glomerulonephritisC. Ischemic infarctionD. TuberculosisE. Acute pyelonephritis

223. Pest’s pneumonia and croupous pneumonia in stage of red hepatization may be differentiated due to presence of following elements in the lungs:A. FibrinB. CarnificationC. MacrophagesD. Red blood cellsE. Edema

224. In the 30yearold woman it was found: cough, sputum with blood, fever, increased blood pressure, decreased urine output, edema of low extremities. All symptoms have developed for 6 weeks. Diagnostic renal biopsy showed Goodpasture’s syndrome. Which pathologic process is characteristic for this syndrome?

A. Autoantibodies to basement membrane of the lungs and kidneysB. Autoantibodies to mitochondrias of the lungs and kidneysC. Autoantibodies to DNAD. Cytotoxic reaction against epithelium of renal tubulesE. Appearance of immune complexes in glomeruli of kidneys and lungs

225. The patient was remove the firmly – elastic, fixed, painful tumor, it is localized in femor. At pathomorphologic research it was found out: yellow tumor without the capsule, greyish with the locuses of necrosis and hemorrhages; histologically is marked the expressed polymorphism of cells. What is your diagnosis? A. Liposarcoma B. Rhabdomyosarcoma C. Angiosarcoma D. Lipoma E. Malignant synovioma

226. Under microscopic examination of the increased cervical lymph node it was found out: the absence of its structure and the lymphoid follicles, infiltration by spherical cells with narrow cytoplasm. It is known from the clinical dates, that other groups of lymph nodules, and lien and liver are increased also. What disease it is necessary to think about? A. Lymphoid leukosis B. Lymphogranulomatosis C. Lymphosarcoma D. Myeloid leucosisE. Myeloma

227. During autopsy of the body of the 30yearsold man has been determined the increased lien (weight 900,0), increased liver (weight 4000,0), increased lymph nodes. Bone marrow of a diaphysis of a femur was red color. Microscopically: infiltration of the portal tract of liver by undifferentiated hemopoietic cells with the spherical shape and narrow cytoplasm. What disease it is possible to think about? A. Chronic lymphoid leucosis B. Chronic myeloid leucosis C. Generalized form of lymphogranulomatosisD. Acute myeloblastic leucosis E. Acute lymphoblastic leucosis

228. At research of a biopsy of the increased cervical lymph node it were found: the absence of structure of lymph nodes, infiltration by proliferated lymphocytes, single Shternberg’s cells. What do these changes testify about? A. Lymphogranulomatosis with predominance of

lymphoid tissue,B. Lymphogranulomatosis with suppression of lymphoid

tissue C. Mixed cellular variant of lymphogranulomatosisD. Nodularsclerotic variant of lymphogranulomatosis E. Lymphosarcoma

229. In biopsy of the increased cervical lymph node of a 14 yearold girl the pathologist has found out: the absence of structure of lymph nodes, the absence of lymphoid follicles, foci of necrosis and sclerosis, cellular polymorphism. There are lymphocytes, eosinocytes, atypical multinuclear cells of the large sizes and uninuclear cells of large sizes

Page 12: Tests General 08

also. Call this disease.A. Lymphogranulomatosis B. Acute lymphoid leucosisC. Chronic lymphoid leucosisD. Berkit’s lymphomaE. Diffuse nonhodgkin’s lymphoma

230. A 14 yearold boy was found out: the increase of lower third of femur, local hyperemia, strengthening venous structure in a zone of pathological process. Ro: injury of supperficial layer of a femoral bone, joint are saved. In biopsy was found out the accumulation of atypical osteoblasts with multiple mitosises and foci of the anomalously formed osteal girders with an invasion in surraunded tissues. Call this disease.A. Osteogenic a sarcomaB. Chondrosarcoma C. Fibrous dysplasia D. Osteoid osteoma E. Osteoblastoclastoma

231. A 40 yearold man has been suffered by a chronic bronchitis has died because of cachexy. During the autopsy it was found: in a lumen of bronchuses the endophytic growth of light grayish soft tissue. Under the microscopic examination in a wall of a bronchus among growths of an atypical squamous epithelium were found the keratinous pearls. What is your diagnosis? A. Squamous carcinoma with a keratinizationB. Squamous carcinoma without a keratinizationC. AdenocarcinomaD. Undifferentiated carcinomaE. Apudoma

232. At intraoperation biopsy of a female brest it was found out: the concentric growth intralobular connective tissue around of smallsized duct covered by a singlelayer epithelium Your presumable diagnosis.A. Fibroadenoma B. Intraductal papillomaC. Lobular carcinoma in sityD. Noninvasive intraductul carcinomaE. Mastitis

233. At gastroscopical research of patient in area of lesser curvature of the stomach it was found out the exophytic formation (1,5 cm in diameter). In biopsy from form it is marked : the growth of atypical glandular structures of different shape and size (stretched, round, with irregular outlines, small or large) into mucosa and submucosa; the epithelial cells are atypical too: their nuclei are polymorphous with various maintenance of chromatin and mitoses. What is your diagnose?A. Adenocarcinoma of stomachB. Squamous carcinoma of stomachC. Polyp of stomachD. Fibrous carcinoma (Skyrrus)E. Chronic ulcer

234. In the skin it was found out well –encapsulated firm node. On the cut it is showed whitish laminated tissue. Histologically: the welldifferentiated connective tissue, the bands of fibers lay in different directions, vessels are distributed irregular. Described morphological features are characteristic for…:

A. FibromaB. RabdomyomaC. PapillomaD. PolypsE. Lipoma

235. In an autopsy of the 60yearold woman pathologist has found out: tumor of uterine with growth surrounded tissue, enlarged regional lymph nodule, hemoperitoneum. Histologically: cellular polymorphism and tissue atypism in the endomtrium, necrosis and hemorrhages, metastasises of adenocarcinoma of lymph nodules. What are secondary appearances of tumors?A. Necrosis and hemorrhageB. Lymphogenous metastasises C. Hematogenous metastasisesD. Cellular polymorphismE. Tissue atypism

236. In young 18yearold woman the sharp pain during swallowing, enlargement of lymph nodes of the neck and increasing of temperature to 39 degrees of Centigrade have appeared. The whitishyellowish pellicles have appeared on the surface of tonsil’s mucosa. After separation of these pellicles the ulceration of tissue was formed. The state of health of the patient has become progressively worse. She has died in 8 days of the disease with signs of progressive heart insufficiency. What histological changes in cardiomyocytes can be found?A. Fatty degeneration;B. Mucousal degeneration;C. Hydropic degeneration;D. Balloon degeneration;E. Hyalinedroplets degeneration.

237. A 55yearold woman has had fever and dyspnea for over a month along with a 2 kg weight loss. On physical examination her temperature is 37.8 C. A chest radiograph shows a reticulonodular pattern along with prominent hilar lymphadenopathy. A transbronchial lung biopsy is performed, and microscopic examination shows no viral inclusions, no fungi, no acid fast bacilli, and no atypical cells. Which of the following diseases is she most likely to have? A. Silicosis B. Sarcoidosis C. Asbestosis D. Tuberculosis E. Usual interstitial pneumonitis

238. A 60yearold man has had a cough without production of much sputum for the past week. On physical examination he is afebrile. There are decreased breath sounds at the right lung base. A chest radiograph reveals an area of consolidation in the right lower lobe. He is given antibiotic therapy, but a month later the radiographic picture has not changed, and his cough continues. A bronchoalveolar lavage is performed and yields atypical cells along with scattered alveolar macrophages. Which of the following is the most likely diagnosis? A. Mycoplasma pneumonia B. Bronchioloalveolar carcinoma C. Sarcoidosis D. Pulmonary infarction E. Silicosis

Page 13: Tests General 08

239. Following a vehicular accident with blood loss leading to prolonged, severe hypotension, a 30yearold man is intubated and placed on a ventilator. He has progressively decreasing oxygen saturations despite increasing PEEP and FI02 of 100%. He remains afebrile. He dies 3 days later. At autopsy, the lungs show diffuse hyaline membranes in alveoli, thickened alveolar walls, and many alveolar macrophages but few neutrophils. Which of the following pulmonary diseases most likely complicated his course? A. Bronchopneumonia B. Chronic bronchitis C. Bronchiectasis D. Viral pneumonia E. Diffuse alveolar damage

240. A 66yearold man has had increasing malaise for the past year. On physical examination auscultation of the chest reveals a friction rub. Laboratory studies show a serum urea nitrogen of 100 mg/dl and creatinine of 9.8 mg/dl. Which of the following forms of pericarditis is he most likely to have? A. Fibrinous B. Hemorrhagic C. Purulent D. Serous E. Constrictive

244. A 45yearold man was rushed to the hospital following the sudden onset of an episode of crushing substernal chest pain. He receives advanced life support measures. His course was marked by intractable cardiogenic shock and he died 4 days later. At autopsy, a large transmural anterolateral area of coagulative necrosis was found in the anterolateral wall of the left ventricle. Which of the following microscopic findings is most likely to be present? A. Fibroblasts and collagen B. Granulation tissue C. Necrotic muscle and neutrophils D. Granulomatous inflammation E. Diffuse chronic inflammation

250. A 41yearold man has a history of drinking 1 to 2 liters of whisky per day for the past 20 years. He has had numerous episodes of nausea and vomiting in the past 5 years. He experiences a bout of prolonged vomiting, followed by massive hematemesis. On physical examination in the emergency room, he has vital signs with T 36.8 C, P 110, RR 22, and BP 80/40 mm Hg. His heart has a regular rate and rhythm with no murmurs and his lungs are clear to auscultation. There is no abdominal tenderness or distension and bowel sounds are present. His stool is negative for occult blood. Which of the following is the most likely diagnosis? A. Esophageal stricture B. Esophageal laceration (MalloryWeiss syndrome) C. Esophageal pulsion diverticulum D. Barrett esophagus (metaplasia with gastric mucosa) E. Esophageal squamous cell carcinoma

251. A 45yearold woman has experienced progressive, increasing muscular weakness, particularly toward the end of the day, over the past 2 months. She does not have arthralgias or myalgias. On physical examination her motor strength goes from 5/5 to 4/5 with repetitive movement of extremities. A chest CT scan reveals an anterior mediastinal mass. Laboratory studies show that her antinuclear antibody

test is negative. By which of the following immunologic mechanisms is her disease most likely produced? A. Local immune complex formation B. Interleukin release from macrophages C. Binding of antireceptor antibody D. Mast cell degranulation E. Crossreactivity with tissue antigens

252. A bee sting is suffered by a 28yearold man, and his wife searches frantically for the medical kit with the injectible epinephrine. Which of the following immunologic mechanisms are they trying to prevent? A. Local immune complex formation B. Interleukin release from macrophages D. Systemic anaphylaxis C. Binding of antireceptor antibody E. Complement activation

253. A 9yearold boy has a sore throat. A throat culture grows group A hemolytic streptococcus. 17 days later develops darkcoloured urine. A renal biopsy was performed. On immunofluoresence staining the biopsy shows granular deposition of IgG and complement around glomerular capillary loops. Which of the following immune hypersensitivity mechanisms is most likely responsible for this pattern of findings? A. Type I B. Type II C. Type III D. Type IV E. Type V

254. A 48yearold man has had a chronic cough with fever for 2 months. On physical examination his temperature is 37.9 C. A chest radiograph reveals a diffuse bilateral reticulonodular pattern. A transbronchial biopsy is performed. On microscopic examination of the biopsy there are focal areas of inflammation containing epithelioid macrophages, Langhans giant cells, and lymphocytes. These findings are most typical for which of the following immunologic responses? A. Type I hypersensitivity B. Type II hypersensitivity C. Graft versus host disease D. Polyclonal Bcell activation E. Type IV hypersensitivity

255. Twelve hours after going on a hike through dense foliage, a 40yearold man notices a slightly raised and tender irregular reddish rash on one forearm that was not covered by clothing. This rash gradually increases in intensity for 2 days and then fades after two weeks. Which of the following forms of hypersensitivity is most likely demonstrated in this patient? A Type I hypersensitivity B Type II hypersensitivity C Type III hypersensitivity D Type IV hypersensitivity E. Type V hypersensitivity

256. The examination of ovarium tumor was performed. On gross inspection of the mass, that was surgically excised, the surface of the mass is smooth, is not adherent to surrounding pelvic structure, and is cystic and filled with hair on sectioning. On microscopic examination there is squamous epithelium, tall columnar glandular epithelium,

Page 14: Tests General 08

cartilage, and fibrous connective tissue. Which of the following neoplasms is she most likely to have? A Teratoma B Choristoma C Hamartoma D Myxoma E Mesothelioma

257. A 44yearold woman who has had multiple sexual partners for the past 30 years has an abnormal Pap smear with cytologic changes suggesting human papillomavirus infection. Without treatment, she is most likely to develop which of the following lesions? A Squamous cell carcinoma B NonHodgkin's lymphoma C Kaposi's sarcoma D Adenocarcinoma E Leiomyoma

258. A 41yearold woman has an exophytic 3 cm mass involving the ectocervix. Pap smear testing is performed; she has never had a previous Pap smear. Cytologic changes that are seen in Pap smears reveals squamous cell carcinoma. Her serum glucose is 157 mg/dL. She has been a commercial sex worker in the past. Which of the following is the most likely risk factor for her cervical carcinoma? A Human papillomavirus infection B Diabetes mellitus, type II C Heavy cigarette smoking D Pelvic inflammatory disease E Previous cancer chemotherapy

259. A 27yearold woman in excellent health has a 2 cm firm, rounded mass is palpable beneath the skin of the left forearm. She has no difficulty using the arm and there is no associated pain with the mass, either in movement or on palpation. The overlying skin appears normal. The mass does not change in size over the next year. Which of the following neoplasms is she most likely to have? A Lipoma B Metastatic carcinoma C Melanoma D Rhabdomyosarcoma E Leiomyosarcoma

260. A 35yearold woman had a firm nodule palpable on the dome of the uterus six years ago recorded on routine physical examination. The nodule has slowly increased in size and is now appears to be about twice the size it was when first discovered. She remains asymptomatic. Which of the following neoplasms is she most likely to have? A Leiomyoma B Adenocarcinoma C Leiomyosarcoma D Hematoma E Metastasis

261. A study is performed to analyze characteristics of malignant neoplasms in biopsy specimens. The biopsies were performed on patients who had palpable mass lesions on digital rectal examination. Which of the following microscopic findings are most likely to indicate that the neoplasm is malignant? A Invasion B Tissue pleomorphism C Tissue atypia

D Increased nuclear/cytoplasmic ratio E Expansive type of growth

262. A clinical study of patient with pharyngeal infections is performed. Patient experienced fever and chills. On physical examination, the most common finding is a pharyngeal purulent exudate. Which of the following types of inflammation did these patient most likely have? A. Acute inflammation B. Granulomatous inflammation C. Abscess formation D. Resolution of inflammation E. Chronic inflammation

263. An empyema is a kind of inflammation as: A Purulent inflammation B Serous inflammation C Fibrinous inflammation D Chronic inflammation E Granulomatous inflammation

264. A 45yearold man has had a fever and dry cough and his temperature is 38.5 C during last 3 days. A chest radiograph shows a right pleural effusion. A right thoracentesis is performed. The fluid obtained has a cloudy appearance with a cell count showing 5500 leukocytes per microliter, 98% of which are neutrophils. Which of the following terms best describes his pleural process? A Purulent inflammation B Serous inflammation C Fibrinous inflammation D Chronic inflammation E Granulomatous inflammation

278. What type of exudates appears in pericardium cavity at a rheumatic pericarditis?

A. Putrefactive B. Serousfibrinous C. HemorrhagicD. Purulent E. Catarrhal

284. A 90yearold woman has pneumonia. Two weeks later a chest radiograph reveals a 3 cm rounded density in the right lower lobe whose liquefied contents form a central airfluid level and has a capsule. Which of the following is the best description for this outcome of her pneumonia? A Abscess formation B Hypertrophic scar C Regeneration D Bronchogenic carcinoma E Progression to chronic inflammation

285. A clinical study is peformed of patients with pharyngeal infections. The most typical clinical course averages 3 days from the time of onset until the patient sees the physician. Most of these patients experienced fever and chills. On physical examination, the most common finding is a pharyngeal purulent exudate. Which of the following types of inflammation did these patients most likely have? A Acute inflammation B Granulomatous inflammation C Abscess formation D Resolution of inflammation E Chronic inflammation

286. At 38 yearold woman it is observed upperclavicle

Page 15: Tests General 08

multiplied lymphatic nodules. During biopsy research in the lymphatic nodules a metastasis of fingerringcellular cancer is exposed. What organ cancer most probably did develop at the patient?A Cancer of stomach B Cancer of thyroidC Cancer of lungsD Cancer of gulletE Cancer of uterus

289. A 70yearold male with a long history of chronic alcoholism has had increasing difficulty with swallowing for the past 2 months. Upper endoscopy reveals an ulcerative midesophageal, 3cm mass that partially occludes the esophageal lumen. He undergoes esophagectomy, and the gross appearance shows ulcerated lesion with heaped up margins and squamous cell atypia of the esophageal mucosa. Identify the type of lesions:A. LeiomyosarcomaB. Squamous cell carcinoma C. Dense collagenous scar D. Adenocarcinoma E. Thrombosed vascular channels

230. An upper gastrointestinal radiographic series reveals gastric outlet obstruction in a 53yearold female who has had nausea, vomiting, and midepigastric pain for several months. Upper endoscopy reveals an ulcerated 3x4 cm mass at the pylorus. The biopsy was performed. Which of the following neoplasms is most likely to be seen on biopsy of this mass? A. NonHodgkin lymphoma B. Neuroendocrine carcinoma C. Squamous cell carcinoma D. Adenocarcinoma E. Leiomyosarcoma

231. A 67yearold male with a long history of Barrett esophagus has had increasing difficulty with swallowing for the past 2 months. In the lower third part of the esophagus upper endoscopy reveals an ulcerative esophageal mass that partially occludes the esophageal lumen. Patient undergoes esophagectomy, and the gross appearance shows ulcerated lesion with necrosis in the center. Identify the type of lesions:A. LeiomyosarcomaB. Squamous cell carcinoma C. Dense collagenous scar D. Adenocarcinoma E. Thrombosed vascular channels

235. A barium swallow is performed in a 44yearold female who has had nausea and vomiting for months. It is known that she had a stomach ulcer for many years. Radiographically, there is marked dilation of the stomach with "beaking" in the distal portion where marked luminal narrowing exists. A biopsy of the antrum shows ulcer formation with elevated margins and prominent submucosal fibrosis without inflammation, epithelium cell atypism. The most likely cause for these findings is A. Chronic duodenum ulcer B. Chronic gastric ulcerC. Acute gastric ulcerD. Gastric carcinoma E. Duodenum carcinoma

236. A 72yearold male with a long history of chronic

gastritis has experienced weight loss and nausea for the past 8 months. He does not have vomiting or diarrhea. In the antrum of stomach endoscopy reveals an ulcerative mass with elevated margins. Patient undergoes gastrectomy, and the gross appearance shows ulcerated lesion with necrosis in the center that has extended below the submucosa, into the muscularis, and has spread more widely extends into the muscularis propria and beyond. Identify the type of lesions:A. Early gastric carcinoma B. Advanced gastric carcinoma C. Duodenum carcinomaD. Acute gastric ulcerE. Chronic gastric ulcer

237. An upper gastrointestinal radiographic series reveals advanced gastric carcinoma in a 53yearold female who has had nausea, vomiting, and midepigastric pain for several months. Ultrasound investigation reveals an solid tumorous enlargements of both ovaries 6x5 cm mass. Identify the type of lesions in ovaries: A. NonHodgkin lymphoma B. Cystadenoma of ovariesC. Virchow's nodeD. Krukenberg tumors E. Cystadenocarcima of ovaries

238. A 67yearold male with a history of brain hemorrhage has had an episodes of hematemesis at the hospital during past 2 weeks after hemorrhage. In the mucous shell of the stomach body endoscopy reveals an ulcer 2.5cm in diameter, round form, with a brown base. Identify the type of ulcer:A. Curling's B. Virchow's C. Chronic D. Cushing's E. Krukenberg’s

240. During autopsy it was found out croupous pneumonia in lower lobe of the left lung and about 500ml of greenishyellow fluid in the pleural cavity. Microscopically pleural liquid contained many neutrophils. Identify the complication of pneumonia:A. Hemothorax B. Fibrinous pleuritisC. PneumothoraxD. Hydrothorax E. Empyema of pleura

241. 56yearold patient has suffered from rightside lowerlobar pneumonia with expectoration of mucus with pus. During autopsy in 910 segments of the right lung the cavity with dense walls filled with purulent masses, was found. The whitish path comes from the cavity toward the radix of the lung. Microscopically it was established that the cavity is separated from notchanged lung tissue with thin membrane, which consists of two layers: internalgranulation tissue, and external – connective tissue. What diagnosis is more probable?A. Chronic abscessB. Pulmonary gangreneC. Acute pulmonary abscessD. Chronic pneumoniaE. Bronchoectatic disease

Page 16: Tests General 08

242. The patient has high temperature, dyspnoe, pain in the right part of the chest. 700 ml of creamy greenyellowish fluid was moved off in the result of pleural puncture. Which of the listed diagnosis is more possible?A. Empiema of pleura B. Hemorragic pleuritisC. Serous pleuritisD. Carcinomatosis of pleuraE. Fibrinous pleuritis

263. At gastroscopical research of patient in area of lesser curvature of the stomach it was found out the exophytic formation (1,5 cm in diameter). In biopsy from form it is marked: the growth of atypical glandular structures of different shape and size (stretched, round, with irregular outlines, small or large) into mucosa and submucosa; the epithelial cells are atypical too: their nuclei are polymorphous with various maintenance of chromatin and mitoses. What is your diagnose?A. Adenocarcinoma of stomachB. Squamous carcinoma of stomachC. Polyp of stomachD. Fibrous carcinoma (Skyrrus)E. Chronic ulcer

271. During the autopsy it was found out the cavity 2,5x1,5 cm within the right frontal part of the brain. It was filled with transparent fluid and its walls were smooth and of brownish colour. What process did develop in the brain?

A. Grey softening of brain.B. Cyst as an outcome of hemorrhage.C. Abscess of brain.D. Birth defect of brain.E. Cyst as a outcome of gray softening

272. In a biopsy of сervix uteri of a26yearold women the diagnosis following was established: pseudoerosion. What microscopical changes has the pathologist revealed? A. Local changes of a stratified squamous epithelium on

singlelayer prismatic one B. Cellatypia of an epithelium of an mucosal epithelium C. Keratinization of an epithelium of an mucosal

epithelium D. “Carcinomatous pearls” E.Local inflammation and necrosis in mucosa

299. In the 30yearold woman systemic lupus erythematosus was diagnosed. Which tissues do the autoantibodies react against?A. Against nuclear constituents of virtually every cell

within the organismB. Only against thyroid constituentsC. Only against basement membrane of kidneys and lungsD. Only against components of bone marrowE. Only against thyroid and cardiac constituents

314. In an experiment, a glass bead is embolized to a branch of the renal artery. A day later there is a focal area in which the renal parenchymal cells in the distribution of the occluded artery show karyolysis and karyorrhexis. The outlines of the cells are still visible, but the nuclei have lost basophilic staining and the cytoplasm is eosinophilic but pale. Which of the following types of cellular necrosis is most likely present? A Coagulative B Caseous C Fatty

D Gangrenous E Liquefactive

315. During hospitalization, a 40yearold woman develops thrombophlebitis. She recovers and is discharged. She returns to her job as an electrician. A couple of months later, which of the following terms would best describe the process seen in a femoral vein after recovery from her thrombophlebitis: A Organization B Acute inflammation C Rupture D Embolization E Propagation

318. Sharp pain at the foot appeared in elderly patient with the atherosclerosis of lower extremities. Left extremity became soft and was increased in volume, skin became blackbrown color with maceration. The demarcation area is absent. Your diagnosis:A Moist gangrene.B Mummification of foot.C Coagulate necrosis of foot.D Dry gangrene.E Sequester of the foot.

326. During autopsy it was found out croupous pneumonia in lower lobe of the left lung & about 500ml of greenishyellow fluid in the pleural cavity. Microscopically pleural liquid contained many neutrophils. Identify the complication of pneumoniaA. Empyema of pleuraB. Fibrinous pleuritisC. PneumothoraxD. Hydrothorax E. Hemothorax

328. At the autopsy of the patient that has suffered for a long time on bronchoectatic disease was found: enlargement of kidneys, they were of the white color & the surface on the cut was lardaceous. Histological investigation: the deposition of homogenous eosinophilic masses colored with Congo red and given of metachromasia with methyl violet color in glomeruli and canals were found. What pathological process took place in the patient?

A. AmyloidosisB. Grainish degenerationC. Fatty degenerationD. Mucoid degenerationE. Hyalinosis

329. Patient died from sudden cardiac death. On the autopsy it was found out the symmetrical type of adipose heart of third degree; the rupture of right ventricle’s wall with hemopericardium and redundant accumulation of fat under epicardium were found in autopsy. Microscopically: the adipose tissue grows from epicardium into myocardium with atrophy of fibers of muscle. What process is more probable?A. Adipose heart;B. Hypertensive disease;C. Ischemic heart disease;D. Fatty degeneration of myocardium;E. Acute myocardial infarction;

Page 17: Tests General 08

330. Death of the patients in early stages of rheumatic fever because of acute cardiac insufficiency is connected with one of the following manifestations of the rheumatic fever:A. Myocarditis B. PericarditisC. EndocarditisD. Septic endocarditisE. Cardiosclerosis

331. The patient died from sudden cardiac death the symmetrical type of adipose heart of third degree; with redundant accumulation of fat under epicardium were found out in autopsy. Microscopically: the adipose tissue grows from epicardium into myocardium with atrophy of fibers of muscle. What process is more probable?

A. Fatty degeneration of myocardiumB. Ischemic heart diseaseC. Acute myocardial infarctionD. Adipose heartE. Hypertensive disease

332. During autopsy of the patient died from chronic cardiac insufficiency the enlarged dense rusty colored lungs with growth of gray color connective tissue around bronchi and vessels were found. How can we call this process in lungs?А Brown induration of the lungВ Hemorrhagic pneumoniaС Interstitial emphysemaD Chronic obstructive emphysemaE Primary idiopathic emphysema;

333.An old patient, who suffered on arteriosclerosis, has been hospitalized in surgical department because he had purulent peritonitis. Thrombosis of mesenteric arteries was found during operation. What is the most probable cause of peritonitis?

A. Hemorrhagic infarctionB. Angiospastic ischemiaC. Angioneurotic edemaD. StasisE. Chronic congestion

338. Biopsy it is:A – surgical removal of organ or extremityB – intravital taking a piece of tissue with subsequent microscopic studyC – taking a piece of tissue with subsequent microscopic study after deathD – dissection of cadaverE – everything is right

339. A sample of thyroid tissue of patient with nodular goiter was taken by means of fine needle aspiration. What type of biopsy was performed?A – puncture biopsyB – endoscopical biopsyC – excision biopsyD – autopsyE – incision biopsy

340. What method of pathomorphological investigation allows to diagnose morphological features of the disease at lifetime, without removal the organ?A – puncture biopsyB – autopsyC – experiment

D – excision biopsyE – surgical removal

341. During gynecological examination of young woman it was found out that she has had cervical erosion. What type of biopsy allows to confirm gynecological diagnosis pathomorphologically?A – endoscopical biopsyB – excision biopsyC – puncture biopsyD – aspiration biopsyE – incision biopsy

342. Patient with gastritis was examined by fibrogastroendoscopy. During the investigation there were taken several pieces of stomach mucosa. What type of biopsy was performed?A – endoscopy biopsyB – excision biopsyC – puncure biopsyD – aspiration biopsyE – incision biopsy

343. Please, choose incorrect statement.A There are extracellular and intracellular accumulations; B Injury can be reversible; C Injury may be caused by hereditary factors; D Injury is characterized by disorder of tissue nutrition and every time leads to total distraction of cells; E Degenerations can appear in one organ or in the whole body

344. Young 18yearold woman died because of the diphtheritic myocarditis with signs of progressive heart insufficiency. What histological changes in cardiomyocytes can be found?A Mucousal degeneration; B Hydropic degeneration; C Fatty degeneration; D Balloon degeneration; E Hyalinedroplets degeneration

345. Prolonged moderate hypoxia create grossly apparent bands of yellow myocardium alternating with bands of darker, redbrown, parts of uninvolved myocardium, such changers in heart called:A “Big yellow heart”; B “Goose heart”; C “Tiger’s heart”; D “Zebra’s heart”; E “Big white heart”

346. Choose one incorrect statement:A. Decomposition – disintegration of membranous

structures of cells and intercellular matrix;B. Infiltration – redundant accumulation of metabolites

into the cells and intercellular matrix; C. The mechanism of decomposition is a base of genetic

storage diseases;D. Perverted synthesis – synthesis of abnormal substances

in the cells and tissues;E. Transformation – formation of one type of metabolism

products from common initial substances for protein, fats and carbohydrates.

347. Intracellular accumulation of fat in liver is characterized by every statement, except one:

Page 18: Tests General 08

A There are small fat vacuoles in cytoplasm around the nuclei of hepatocytes; B Fat is accumulated in extracellular spaces; C Liver enlarged and becomes yellow; D Its consistency is soft and greasy; E Fat is stained by special stain Sudan IV, and becomes black color

348. Morphological changers in liver that is called “goose liver” means:A intracellular accumulation of lipids in liver cells; B extracellular accumulation of lipids in liver cells;C intracellular accumulation of glycogen in liver cells; D extracellular accumulation of glycogen in liver cells; E intracellular accumulation of proteins in liver cells

349. Cellular swelling of kidney tubular epithelium at protracted hypoxia can lead to:A intracellular accumulation of proteins; B intracellular accumulation of lipids; C intracellular accumulation of glycogen; D focal colliquative necrosis of epithelium; E accumulation of lipids in kidney’s stroma

350. What etiologic factors can be most influential at the development of intracellular accumulation of lipids in liver? A. Insufficiency of vitamins B. Pathology of protein exchange C. Pathology of pigment exchange D. Influence of hepatotoxic substances E. Pathology of glycogen exchange

351. In epithelium of kidney tubules at disorders producing proteinuria pinocytotic reabsorption of the protein form drops, which appear pink color in hematoxylin and eosin staining, it is:A Balloon degeneration; B Mucousal degeneration; C Hydropic degeneration; D Fatty degeneration; E Hyalinedroplets degeneration;

352. Mechanism of intracellular accumulation of proteins in epithelium of kidney tubules at disorders producing proteinuria is: A Infiltration, B Decomposition, C Perverted synthesis, D Transformation; E Phanerosis

353. The ultrastructural changes at cellular swelling consist of the following, except one: A blebbing, blunting, and distortion of microvilli at plasma membrane; B swelling, rarefaction, and the appearance of phospholipidrich amorphous densities in mitochondria; C dilatation of the endoplasmic reticulum with detachment and disaggregation of polysomes; D nucleolar alterations; E pinocytotic vesicles are full with proteins

354. At patient with diabetes mellitus it is observed: increased in size liver, increased levels of triglyceride and lipoproteid’s in blood. What type of dystrophy can be expected in a liver? A. Intracellular fatty dystrophy B. Cellular swelling

C. Intracellular protein dystrophy D. Hyalinosis E. Extracellular carbohydrate dystrophy

355. A punctual biopsy of liver at patient with diabetes mellitus that has suffered about 7 years was performed. Microscopically there can be found changers of liver parenchyma as:A small fat vacuoles in the cytoplasm; B fat vacuoles are situated around the nucleus of hepatocytes; C hepatocytes are increased in size; D – everything is right; E – everything is wrong

356. A 40yearold male was diagnosed with an undifferentiated carcinoma of the lung. Despite treatment with chemotherapy, he died of widespread metastases. At autopsy, tumor was found in many organs. Histologic examination revealed many foci in which individual tumor cells appeared shrunken and deeply eosinophilic. Their nuclei showed condensed aggregates of chromatin under the nuclear membrane. The process affecting these shrunken tumor cells was triggered by the release of which of the following proteins into the cytosol? A. Lipofuscin B. Cytochrome C C. Catalase D. Phospholipase E. bcl2

357. Many drugs that are used to treat cancer cause death of tumor cells by apoptosis. Mutational inactivation of which of the following genes can render tumor cells resistant to the effects of such chemotherapeutic drugs? A. bcl2 B. p53 C. NFkB D. P450 E. Granzyme B

362. Thickened, intertwisted and stony dense aortic valve was discovered at the autopsy of a 72yearold male. The heart weighed 580g, with marked left ventricular hypertrophy and minimal coronary arterial atherosclerosis. A serum chemistry panel revealed no abnormalities prior to death from congestive heart failure. Which of the following pathologic processes accounts for the appearance of the valve? A. Amyloidosis, B. Dystrophic calcification, C. Lipofuscin deposition, D. Hemosiderosis, E. Fatty change

363. Metastatic calcification is most likely to occur in which of the following conditions? A. Tuberculosis of the lung, B. Acute hemorrhagic pancreatitis, C. Aortic stenosis in a 70yearold man, D. Vitamin D intoxication, E. Amyloidosis

364. Deposition of calcium in the renal tubular epithelium in patients with primary hyperparathyroidism is the result of which of the following processes? A. Dystrophic calcification,

Page 19: Tests General 08

B. Renal tubular atrophy, C. Autophagocytosis, D. Metastatic calcification, E. Cellular aging

365. A 35yearold man struck on his leg by a falling pallet rack in the region of thigh. The skin is not broken. Within 2 days there is a 5x7cm purple color to the site of injury. Which of the following substances has most likely accumulated at the site of injury to produce a yellowbrown color 16 days after the injury? A. Lipofuscin, B. Bilirubin, C. Melanin, D. Hemosiderin, E. Glycogen

366. At autopsy, the heart of a 63yearold male is only 250gr, with small right and left ventricles. The myocardium is firm, a yellowishbrown color. The coronary arteries demonstrate very little atherosclerosis. Which of the following substances most likely can be found in the myocardial fibers of this heart? A. Melanin B. Hemosiderin C. Glycogen D. Lipofuscin E. Bilirubin

367. A 22yearold female has a congenital anemia that required multiple transfusions of red blood cells for many years. She now has no significant findings on physical examination. However, her liver function test results are abnormal. Which of the following findings would most likely appear in a liver biopsy? A. Steatosis in hepatocytes B. Bilirubin in canaliculi C. Glycogen in hepatocytes D. Amyloid in portal triads E. Hemosiderin in hepatocytes

368. Accumulation of lipofuscin granules in cells is typically seen in which of the following conditions? A. Atrophy B. Hypertrophy C. Hyperplasia D. Metaplasia E. Apoptosis

369. A fight carotid endarterectomy is performed on a 69yearold female who had an audible bruit on auscultation of the neck. Examination of the curetted atheromatous plaque reveals a grossly yellowtan, firm appearance. Microscopically, which of the following materials can be found in abundance in the form of crystals producing long, cleftlike spaces? A. Glycogen B. Lipofuscin C. Hemosiderin D. Immunoglobulin E. Cholesterol

370. In patient with jaundice the following data were established: in serum the increasing of bilirubin because of the unconjugated form; in faeces and urine increasing of stercobilin; the level of conjugated (direct) bilirubin in serum is normal. What type of jaundice takes place?

A. Jaundice of newborns; B. Parenchymatous (hepatic) jaundice; C. Haemolytic jaundice; D. Gilbert’s disease; E. Mechanical (posthepatic) jaundice

371. A 53yearold patient with insufficiency of mitral valve has a cough with ferruginous sputum. What pigment has caused such color of sputum? А. Hematoidin, В. Melanin, С. Hemoglobin, D. Hemomelanin, Е. Hemosiderin

372. Hemolysis developed after the bite of snake at a young girl. Jaundice of skin, sclera and mucous was observed. Accumulation of what pigment is the cause of jaundice? А. Direct bilirubin В. Indirect bilirubin С. Direct and indirect bilirubin D. Ferritin Е. Hemomelanin

373. 77yearold patient suddenly died because of heart attack of myocardium. During autopsy in heart it was found out: thrombosis of the left anterior descending coronal artery and a focus of yellowish softening with hemorrhagic halo in the left ventricle anterior wall. Which of the following patterns of tissue injury was most likely? A. Liquefactive necrosis B. Caseous necrosis C. Coagulative necrosis D. Fat necrosis E. Gangrenous necrosis

374. A 68yearold female suddenly lost consciousness and, on awakening an hour later, could not speak or move her right arm and leg. Two months later, a head computed tomography (CT) scan showed a large cystic area in her left parietal lobe. Which of the following pathologic processes most likely occurred in the brain? A. Fat necrosis B. Coagulative necrosis C. Apoptosis D. Liquefactive necrosis E. Karyolysis

375. The pain has appeared in the left foot of 57yearold patient suffered with diabetes mellitus. After some days the foot became enlarged in size with soft, swollen edematous, putrid, black color and macerated skin; the demarcation line is not clear. What pathological process takes place in a patient? A. Coagulative necrosis B. Sequestrum C. Dry gangrene D. Wet gangrene E. Noma

376. On day 28 of the menstrual cycle in a 23yearold female, there is menstrual bleeding that lasts for a few days. She has had these regular cycles for many years. Which of the following processes is most likely happening in the endometrium just before the onset of bleeding? A. Apoptosis B. Caseous necrosis

Page 20: Tests General 08

C. Heterophagocytosis D. Atrophy E. Liquefactive necrosis

377. A renal biopsy is performed on a 33yearold female who has had increasing renal failure for the past week. Which of the following changes seen with electron microscopy most likely suggests a diagnosis of acute tubular necrosis? A. Mitochondrial swelling B. Plasma membrane blebs C. Chromatin clumping D. Nuclear fragmentation E. Ribosomal disaggregation from endoplasmic reticulure

378. In 87yearold patient suffered with atherosclerosis the pain has appeared in the right foot. After several days the foot decreased in size, skin became dark black color that resembled the foot of mummy, the demarcation line was clear. Which of the following patterns of tissue injury was in patient? A. Coagulative necrosis B. Sequestrum C. Dry gangrene D. Wet gangrene E. Noma

379. A 73yearold man had a "stroke." A cerebral angiogram demonstrates occlusion of the left middle cerebral artery. Which of the following pathologic alteration from this event that occurs in pathient’s brain is the most likely? A. Cerebral softening from liquefactive necrosis B. Infarction with coagulative necrosis C. Predominant loss of glial cells D. Recovering of damaged neurons E. Wet gangrene with secondary bacterial infection

380. In an experiment, a glass bead is embolized to a branch of the renal artery. A day later there is a focal area in which the renal parenchymal cells in the distribution of the occluded artery show karyolysis and karyorrhexis. The outlines of the cells are still visible, but the nuclei have lost basophilic staining and the cytoplasm is eosinophilic but pale. Which of the following types of cellular necrosis is most likely present? A. Coagulative B. Caseous C. Fatty D. Gangrenous E. Liquefactive

381. The cavity 3,5x2,5 cm was found out during the autopsy in the right frontal part of the brain. It was filled in with yellowishwhitish fluid, cavity walls were smooth and the same color as brain tissue. Name the process that is developed in the brain? A. Cyst as an outcome of hemorrhage B. Abscess of brain. C. Hemorrhagic infarction of brain. D. Birth defect of brain. E. Cyst as an outcome of gray softening

382. Choose the correct statement that can characterize apoptosis: A. It is programmed cell’s death in the living organism with fragmentation of cell into apoptotic bodies at the end the process

B. It is the premature death and destruction of cell’s organelles in the living organism C. It is cell’s death in the living organism under action of critical damage factors D. It is postmortem destruction of cells under action of the bacteria’s enzymes E. It is destruction of all components of organs under action of critical alteration

383. During electronic microscopic investigation of brain tissue it was observed cells with chromatin aggregations in the peripheral zones, under the nuclear membrane, some nucleus break up, producing two or more fragments, some cells with extensive surface blebbing, that undergoes fragmentation into a number of membranebound parts. What pathological process was seen in the cells? A. Autolysis B. Necrosis C. Phagocytosis D. Apoptosis E. Necrobiosis

384. A 40yearold male was diagnosed with an undifferentiated carcinoma of lung. Despite treatment with chemotherapy, he died of widespread metastases. At autopsy, tumor was found in many organs. Histologic examination revealed many foci in which individual tumor cells appeared shrunken and deeply eosinophilic. Their nuclei showed condensed aggregates of chromatin under the nuclear membrane. What process does affect these shrunken of tumor cells? A. Autolysis B. Necrosis C. Phagocytosis D. Apoptosis E. Necrobiosis

385. An elderly patient had an acute violation of cerebral blood circulation that was accomplished with coma and recent patient’s death. During autopsy a large cavity filled with blood was observed in the right hemisphere of brain. What pathological process was found out in the right hemisphere of brain? A. Haematoma B. Hemorrhagic infiltration C. Heart attack of brain D. Diapedetic hemorrhage E. Tumor of brain

386. A 55yearold female has had discomfort with swelling of her left leg for the past week. The leg is slightly difficult to move, but there is no pain on palpation. A venogram reveals thrombosis of deep left leg veins. Which of the following conditions take place? A. Haematoma B. General congestion C. Local hyperemia D. Local congestion E. General hyperemia

387. During autopsy it was found out an increased in size liver, the picture of liver parenchyma on the cut look like muscatel nuts. In hepatic vein nearwall blood clots were found out (BaddaKiary syndrome). Name the type of violation of blood circulation in liver. A. General hyperemia B. General congestion

Page 21: Tests General 08

C. Local congestion D. Local hyperemia E. Haematoma

388. A 56yearold diabetic male presented with leftsided chest pain that radiated to the arm. Coronary angiography revealed occlusion of the left anterior descending artery. After some days developed severe breathlessness and was diagnosed to have acute left ventricular failure. Which of the following histological changes in lungs would most likely observed? A. Congestion of alveolar capillaries with fibrin and neutrophils in alveoli B. Congestion of alveolar capillaries with transudate in the alveoli C. Fibrosis of alveolar walls with heart failure cells in alveoli D. Multiple areas of hemorrhages E. A purulent exudate in the pleural space

389. A 32yearold patient with chronic ulcer has had a vomiting by blood. Name this process: A. Epistaxis, B. Haemotenesis, C. Melaena, D. Metrorrhagia, E. Hemotorax

390. A 76yearold female fell and fractured her left femoral trochanter. After 2 weeks in the hospital, her left leg is swollen. She experiences pain on movement of this leg, and there is tenderness to palpation. Angiography revealed deep venous thrombosis. Which of the following complications is most likely to occur in the leg?A. Local hyperemia B. Local congestion C. General congestion D. General hyperemia E. Haematoma

391. After falling in the bathtub and striking her head, a 78yearold female becomes increasingly somnolent. A day later, a head computed tomography scan demonstrates an accumulation of fluid beneath the dura, compressing the left cerebral hemisphere. What is the best term for this fluid collection? A. Hematoma B. Purpura C. Congestion D. Petechia E. Ecchymosis

392. Transmural myocardial infarction in the 66yearold patient was complicated with edema of the lungs, formation of transudes in cavities (hydrothorax and hydropericardium). What is the type of heart insufficiency? A. Cyanosis, B. Right ventricle insufficiency, C. Arterial hypertension, D. General hyperemia, E. Progressive left ventricle insufficiency

393. A 69yearsold woman suffered from atherosclerosis and died due to cardiacpulmonary insufficiency. At autopsy features of congestion in parencymal organs, hypertrophy of left ventricle heart were found out. Which of the following macroscopical changes in liver parenchyma

would most likely observed? A. “Goose liver”. B. “Big white liver”. C. Cirrhosis. D. “Nutmeg liver”. E. Chronic hepatitis.

394. Thickened, intertwisted and stony dense aortic valve was discovered at the autopsy of a 72yearold male. The heart weighed 580g, with marked left ventricular hypertrophy and postinfarction scaring of left ventricle wall. Acrocyanosis, edema of subcutaneous tissues, hemosiderosis of lungs and pneumosclerosis, depositing of blood in the parenhymal organs were observed. Which of the following pathologic processes took place? A. Congestive heart failure, B. Acute right ventricle insufficiency, C. Arterial hypertension, D. General hyperemia, E. Acute left ventricle insufficiency

395. During histological investigation of lymphatic nodules at patient with hypogammaglobulinemia, generally affecting all the AB classes and normal levels of circulating Bcells it was observed hyperplasia of the Bcell areas of lymphoid follicles in the nodes. What is your diagnose?A. Severe Combined Immunodeficiency; B. DiGeorge's Syndrome; C. Bruton's Disease; D. Common Variable Immunodeficiency; E. WiskottAldrich Syndrome

396. 56yearold patient has suffered bronchoectatic disease and hemoptysis, the edema of face and waist has appeared. The protein (33 mg/l) was found in urine. Pulmonary hemorrhage was the cause of patient’s death. At autopsy it was found enlargement of kidneys, on the section they were dense with lardaceous surface. Histologically: the deposition of homogenous eosinophilic masses colored with Congo red in orange and given of metachromasia with methyl violet color in glomeruli and canals were found. What pathological process is described?A. Hyalinosis; B. Grainish degeneration; C. Fatty degeneration; D. Mucoid degeneration; E. Amyloidosis

397. Histologic investigation of thyroid gland has showed destruction and atrophy of follicles, diffuse lymphoid infiltration with formation of lymphoid follicles in the stroma. Which of the following disease’s type is described?A. Autoimmune specific; B. Autoimmune nonspecific; C. Bacterial; D. Infectiousallergic; E. Viruses infectious

398. Which of the following diseases is not organospecific autoimmune disease in origin? A. Hashimoto's thyroiditis; B. Autoimmune hemolytic anemia; C. Goodpasture's syndrome; D. Systemic Lupus Erythematous; E. Graves' disease

399. Which of the following diseases can be characterized

Page 22: Tests General 08

as an Xlinked recessive disease with eczema, thrombocytopenia, and recurrent infections?A. Severe Combined Immunodeficiency; B. DiGeorge's Syndrome; C. Bruton's Disease; D. Common Variable Immunodeficiency; E. WiskottAldrich Syndrome

400. At 46yearold patient with systemic lupus erythematous during histological investigation of skin biopsy it was observed vasculitis and dermal chronic inflammatory infiltrates. What type of the disease is revealed? A. Autoimmune specific; B. Autoimmune nonspecific; C. Bacterial; D. Infectiousallergic; E. Viruses infectious

401. Inadequacy of immune answer because of an innate defect in the immune system (defect of histogenesis of immunocytes, violation of thymus embryogenesis or regulation of the immune system) is named as: A. Primary immunodeficiency syndrome; B. Secondary immunodeficiency syndrome; C. Autoimmune specific; D. Autoimmune nonspecific; E. Infectiousallergic

402. At 2yearold patient recurrent bacterial infections such as pharyngitis, sinusitis, bronchitis and pneumonia were observed during past 10 months. The causative organisms were Haemophilus influenzae and Staphylococcus aureus. The failure of preB cells differentiation into mature B cells was distinguished. Identify the type of disease: A. Severe Combined Immunodeficiency; B. DiGeorge's Syndrome; C. Bruton's Disease; D. Common Variable Immunodeficiency;E. WiskottAldrich Syndrome

403. During autopsy of infant that died because of lung fungal infections it was observed rudimentary thymus and absence of Tcells. The serum levels of all classes of immunoglobulins and amount of Bcells were normal. Which of the following disease took place in patient? A. Severe Combined Immunodeficiency; B. DiGeorge's Syndrome; C. Bruton's Disease; D. Common Variable Immunodeficiency; E. WiskottAldrich Syndrome

404. A 48yearold patient has had fibrouscavernous tuberculosis. The increasing of protein, presence of hyaline, grain cylinders and erythrocytes were found in urine. The patient has died in a month because of kidney insufficiency. At autopsy the enlargement of the heart and "lardaceus" kidneys were found. What is the complication of fibrosecavernouse tuberculosis?A. Nephrotic syndrome; B. Аmyloidosis; C. Hyalinosis; D. Grainish degeneration; E. Mucoid degeneration

405. During postmortem investigation of 1,5yearold child it was observed: decreased amount of Bcells, the serum levels of all classes of immunoglobulins were depressed, germinal

centers of lymph nodes, Peyer's patches, and tonsils were underdeveloped, absence of plasma cells throughout the body. Which of the following disease took place in patient? A. Severe Combined Immunodeficiency; B. DiGeorge's Syndrome; C. Bruton's Disease; D. Common Variable Immunodeficiency; E. WiskottAldrich Syndrome

406. A clinical study of patient with pharyngeal infections is performed. Patient experienced fever and chills. On physical examination, the most common finding is a pharyngeal purulent exudate. Which of the following types of inflammation did these patient most likely have? A. Resolution of inflammation, B. Granulomatous inflammation, C. Abscess, formation, D. Acute inflammation, E. Chronic inflammation

407. A 43yearold man has had a fever and dry cough and his temperature is 38.5 C during last 3 days. A chest radiograph shows a right pleural effusion. A right thoracentesis is performed. The fluid obtained has a cloudy appearance with a cell count showing 5500 leukocytes per microliter, 98% of which are neutrophils and high level of proteins. Which of the following terms best describes his pleural process? A. Granulomatous inflammation, B. Serous inflammation, C Fibrinous inflammation, D. Catarrhal inflammation, E. Purulent inflammation

408. A 90yearold woman has pneumonia. Two weeks later a chest radiograph reveals a 3cm rounded density that contains liquid with capsule in the right lower lobe. During punctual biopsy there was obtained 100ml of yellowishcloudy fluid. Which of the following is the best description for this outcome of her pneumonia? A. Abscess formation B. Phlegmon, C. Furuncle, D. Carbuncle, E. Empyema

409. The patient has suffered from rightside lowerlobar pneumonia with expectoration of mucus with pus. In autopsy in 910 segments of the right lung the cavity with dense walls filled with purulent masses, was found. The whitish path comes from the cavity toward the radix of the lung. Microscopically in was established that the cavity is divided from saved lung tissue with thin membrane, which consists of two layers: internalgranulation tissue, and external – connective tissue. What diagnosis is more probable?A. Empyema, B. Pulmonary gangrene, C. Acute pulmonary abscess, D. Phlegmon, E. Chronic abscess

410. A 22yearsold patient was admitted into the hospital with heavy nasal breathing. During the examination of nasal cavity it was found out the thickened mucous membrane, a lot of mucus and nodular infiltrates without erosions. It was diagnosed as rhinoscleroma of nose. Biopsy was performed.

Page 23: Tests General 08

What typical morphological changes may be found? A. Granulomas with Virchow’s cells, B. Granulomas with Mikulicz’s cells, C. Granulomas with Langhan’s cells, D. Granulomas with foreign body cells, E. Interstitial inflammation

411. A man has died in 8th day since beginning of the disease. It was diagnosed dysentery. During the autopsy it was found out a thickened wall of the sigma and rectum, fibrinous membrane on the surface of mucous membrane. Histologically: there is a deep necrosis of mucous membrane with infiltration of necrotic masses with fibrin. What kind of colitis does correspond to those changes? A. Catarrhal, B. Ulcerative, C. Diphtheritic, D. Chronic, E. Gangrenous

412. After mistaking using of some toxic substance an acute toxic necrosis of liver has developed at 35yearold male. All necrotic mass of liver was surgically removed. Few months later the total mass and normal structure of liver was restored. What kind of reparation was represented in liver? A – restitution, B – substitution, C – physiological regeneration, D – hyperregeneration, E – metaplasia

413. In the section of 70yearold male who had died from cardiovascular insufficiency it was found an area of sclerosis in myocardium. What type of regeneration has developed in myocardium after myocardial infarction? A – physiological regeneration, B – substitution, C – restitution, D – hyperregeneration, E – hyporegeneration

414. During cooking young woman suddenly cut her finger by knife. She immediately took all necessary measures and covered a wound by aseptic bandage. 2 weeks later the wound was healed by first intension. Choose the one which is not correct for healing by first intension: A – wound is clean and uninfected, B – wound is at times infected, C – wound is surgically incised, D – without much loss of cells and tissue, E – edges of wound are approximated by surgical sutures

415. Regeneration of endometrium at the beginning of each physiological ovarialmenstrual cycle at women of reproductive period is an example of: A – restitution, B – hyperregeneration, C – physiological regeneration, D – substitution, E – metaplasia

416.Microscopical investigation of cardiac muscle around sclerosis area after infarction had found that myocardiocytes and their nuclei were enlarged in size and had features of increased functional capacity. What process developed in myocardium around sclerosis area? A – hyperplasia of myocardiocytes,

B – hypertrophy of myocardiocytes, C – atrophy of myocardiocytes, D – metaplasia of myocardiocytes, E – hypoplasia of myocardiocytes

417. During microscopical examination of operative gynecological material of 45yearold woman with clinical diagnosis of fibromyoma uteri it was found that endometrium was thickened up to 1 sm, had a lot of glands of different shapes, sizes and localization. What process was performed in endometrium? A – endometrium hyperplasia, B – endometrium hypertrophy, C – endometrium atrophy, D – endometrium metaplasia, E – endometrium carcinoma

418. Biopsy of upper respiratory tract tissues taken from 45yearold man who smokes during 25 years has revealed that columnar mucussecreting respiratory epithelium was replaced by squamous epithelium. What kind of adaptation in epithelium was performed? A –hypertrophy, B –metaplasia, C – atrophy, D – hypoplasia, E – hyperplasia

419.In the section of 76yearold woman it was found that her ovari were small and dense. Microscopically they were represented by prevailing connective tissue. What process has happened in ovari of this woman? A – physiological atrophy, B – pathological atrophy, C – aplasia, D – metaplasia, E – hypotrophy

420. In the section of 65yearold man who died from septic endocarditis it was found that aortic valve outflow was severely narrowed by disease and the muscle of left ventricle of the heart was thickened up to 2 sm. What process of adaptation was performed in cardiac muscle of left ventricle? A – involution of myocardium, B – myocardiac hyperplasia, C – myocardiac hypotrophy, D – myocardiac hypertrophy, E – atrophy

421. Macroscopical study of skin formation has found that it looks like bush of branching papillae, the same color as skin; microscopically it is represented by connective tissue stem covered by epithelium like with a glove. What type of neoplasia is observed? A – adenoma, B – papilloma, C – sarcoma, D – skin cancer, E – adenocarcinoma.

422. Microscopical study of mucosal formation from nose cavity showed that it was represented by loose, swollen stroma, consisting of thin fibers with thinwalled vessels and covered with ciliated epithelium. What type of neoplasia may be supposed?A – soft papilloma,

Page 24: Tests General 08

B – adenoma, C – hard papilloma, D – cancer “in situ”, E – adenocarcinoma.

423. Microscopical study of ovarium has found out enlarged cavity covered by cylindric epithelium inside and filled with serous fluid. What type of formation has been found in ovarium? A – cystoadenoma, B – adenocarcinoma, C – white body, D – cyst of yellow body. E – normal folicullus.

423. Microscopical study of biopsy taken from stomach has revealed growth of glandular atypical cells with submucosal invasion. What type of neoplasia has been found? A – early gastric carcinoma, B – advanced gastric carcinoma, C – adenoma, D – soft papilloma, E fungating carcinoma.

424. 42yearold man suddenly found out enlarged round formation at the left supraclavicular region. Biopsy and further histological study showed that it was lymphnode with blood and lymphatic vessels full of tumor cells. What process is performed? A – metastasis called Virchov’s gland, B – Shnitsler’s metastasis, C – Krukenberg’s metastasis, D – Paget’s disease, E – Wilms tumor

425. Histological study of node of the thyroid gland found out encapsulated growth of atypical cells with capsula's invasion. What process has developed in the thyroid gland? A – Hashimoto's thyroiditis, B – follicular adenocarcinoma, C – Grave's disease, D – nodular colloid goiter, E – Bcells adenoma of the thyroid gland

426. Microscopical study of skin biopsy taken from unhealing ulcer of the leg found out nestlike accumulations of atypical cells of multilayer squamous epithelium with characteristic “pearls”. What is the type of tumor? A – adenocarcinoma, B – colloid cancer, C – solid cancer, D – squamouscell kratinishing cancer of skin, E – scirrhous cancer

427. Macroscopical study of surgically removed esophagus found out annularlike stenosing narrowing of its lumen due to infiltrative growth into the wall of esophagus. Microscopically it was established squamouscell carcinoma of esophagus. What type of carcinoma growth was represented? A – polypoid fungating type, B – ulcerating type, C – diffuse infiltrating type, D – adenoid cystic type, E – intestinal type

428. 35yearold woman paid attention that skin of the left nipple and areola had become crusted, fissured and ulcerated with oozing of serousanguineous fluid from the erosions. Histological study of tissues taken from lesion area found out thickened squamous multilayered epithelium and enlarged spherical cells with hyperchromatic nuclei with cytoplasmic halo, stained positively with mucicarmine, it also was found invasive duct carcinoma. What type of breast tumor is performed? A – colloid carcinoma, B – lobular carcinoma “in situ”, C – Paget’s disease, D – carcinoma noninvasive, E – infiltrating ductal carcinoma – NOS

429. 56yearold man who had been smoking during 35 years was operated because of neoplasia in the left lung. Histological study of removed tissue revealed in the area of neoplasia enlarged cells with large nuclei, prominent nucleoli, abundant cytoplasm and welldefined cell borders. Tumor cells lacked specific features that could assigned them into squamouscell carcinoma or adenocarcinoma. What kind of tumor of lung may be supposed? A – large cell carcinoma, B – small cell carcinoma, C – oatcell carcinoma, D – papillary adenocarcinoma, E – combined oatcell carcinoma.

430.Macroscopical study of surgically subtotal removed stomach has revealed localized growth in pyloric antrum represented by thickened stomach wall due to extensive desmoplasia giving the appearance as “leather bottle stomach”. What type of stomach cancer may be supposed? A – scirrhous carcinoma, B – colloid carcinoma, C – fungating carcinoma, D – ulcerative carcinoma, E – ulcercancer

431. In the autopsy of 46yearold woman it was found tumor nodes in both enlarged ovaries. Histological study established “Krukenberg’s” cancer of ovaries. What is the primary localization of basic tumor growth? A – lung, B – bone, C – urinary bladder, D – breast, E – stomach

432. Histological study of endometrium of 68yearold woman has revealed abundant quantity of abnormal glands with papillar structures formed by atypical cells. What process is performed? A – hyperplasia of endometrium, B – adenocarcinoma of endometrium, C – soft papilloma of endometrium, D – metaplasia of endometrium, E – atrophy of endometrium

433. Histological study of biopsy pattern of stomach tissues has found enlarged oval malignant cells, optically empty at hematoxilineosin staining, but SHIKpositive cytoplasm with peripheral localization of nuclei. What type of gastric carcinoma may be supposed? A – papillary adenoma, B – papillary adenocarcinoma,

Page 25: Tests General 08

C – signetring cell carcinoma, D – scirrhous carcinoma, E – solid carcinoma

434. At bronchoscopy it was found malignant tumor growth into lumen of bronchi. What kind of tumor growth takes place in relation to the lumen of bronchi? A – expansive growth, B – endophytic growth, C – exophytic growth, D – invasive growth, E – apposition growth

435. During dinner young boy was in a hurry and took a spoon of very hot soup thus made a burn of mucosa of tongue. Few days later no marks of burn were found. Epithelial tissue of tongue return to norm. What process was performed in epithelial tissue of tongue? A – reparative regeneration, B – substitution, C – metaplasia, D – physiological regeneration, E – pathological regeneration

436. 23yearold woman paid attention at appearance of nodular formations in her breast after 6months using of hormone contraceptives without doctor’s control. What process may be supposed in breast? A – nodular hypertrophy, B – nodular hyperplasia, C – nodular metaplasia, D – diffuse hyperplasia, E – nodular atrophy

437. 67yearold patient suffered from arterial hypertension during 25 years. Progress of cardiovascular insufficiency leaded to death. In autopsy it was found that heart was increased in size, its chambers were dilated, myocardium of heart wall was thickened up to 2,5 sm. What kind of changes were found in heart? A– eccentric hypertrophy, B – concentric hypertrophy, C – myocardial hyperplasia, D – involution of myocardium, E – myocardial hypotrophy

438. 25yearold person had diagnosed fracture of left femoral bone. What kind of changes in skeletal muscles of left leg may developed after 1 month of immobilization? A – denervation atrophy, B – atrophy due to physical influence, C – ischemic atrophy, D – atrophy due to pressure, E – disuse atrophy

439. A patient with arterial hypertension has clinical manifestation of cardiovascular insufficiency with hemodynamic deterioration. What kind of changes in myocardium may be found? A – concentric hypertrophy, B – eccentric hypertrophy, C – myocardial hyperplasia, D – myocardial atrophy, E – myocardiac hyperplasia

440. Young man got into accident and had multiplied trauma. Serious damage of one of the kidney leaded to its

surgical removal. Some time later it was found that remained kidney had increased in size and weight. What has happened with remained kidney? A – tumor hyperplasia, B – compensatory hypertrophy, C – compensatory hyperplasia, D – metaplasia, E – dysplasia

441. A 11yearsold child was admitted into the hospital with complaints of fever, weakness, and intense pain in the right thigh. It was defined that he has bitten by dog 3 days ago. During the visual examination it was found out redness, swelling and acute painfulness along right thigh. After excision it was made the histological examination, which was showed a considerable accumulations of polymorphonucleus leukocytes between the mussel fibers with purulent fusion. Which of the following terms best describes such process? A. Abscess, B. Phlegmon, C. Gangrene, D. Empyema, E. Croupous inflammation

442. A 27yearsold woman has been suffered diabetes mellitus since she had been a child. During the last years the arterial hypertension and proteinuria have occurred. She has died with signs of uremia. During autopsy it was found out increased in size heart and fibrinous membrane on the surface of epicardium and pericardium. Which of the following terms best describes such process in the heart? A. “Cor villosum”, B. “Tiger heart”, C. “Armor heart”, D. Adipose heart, E. Purulent pericarditis

443. A 44yearsold patient has a burn of right hand. The exfoliation of epidermis and formation of bubbles filled by semitransparent fluid is manifested in the palm and the back surface of the hand. What kind of inflammation does occur in that case? A. Purulent, B. Fibrinous, C. Putrefactive, D. Serous, E. Catarrhal

444. A patient was admitted in the hospital with diagnosis of trichinellosis. What kind of inflammation develops within the muscles around the parasites? A. Exudative fibrinous, B. Exudative purulent, C. Proliferative granulematous, D. Exudative serous, E. Proliferative interstitial

445. A 1/3 of the liver with a multicellular echinococcus was resected during the operation. What typical changes in the liver tissue surrounding the parasites were found out by the pathologist during the histological examination of the resected part of the organ? A. Formation of noninfectional granulomas, B. Flabby consistence of the liver, C. Exudative tissue reaction, D. Formation of infectional granulomas,

Page 26: Tests General 08

E. Hydropic dystrophy

446. A 43yearold male has experienced midepigastric abdominal pain for the past 3 months. Upper endoscopy reveals a 2cm ulceration of the gastric antrum. A biopsy of the ulcer shows angiogenesis with fibrosis and mononuclear cell infiltrates with lymphocytes, macrophages, and plasma cells. The best term for this pathologic process is: A. Acute inflammation, B. Serous inflammation, C. Granulomatous inflammation, D. Fibrinous inflammation, E. Chronic inflammation

447. A month after an appendectomy, a 25yearold woman palpates a small nodule beneath the skin at the site of the healed right lower quadrant incision. The nodule is excised and microscopically shows macrophages, collagen, a few small lymphocytes, and giant cells. Polarizable, refractile material is in the nodule. Which of the following complications of her surgery best accounts for these findings? A. Chronic inflammation, B. Abscess formation, C. Foreign body granuloma, D. Ulceration, E. Edema

448. A 45yearold male has had a cough with fever for the previous 2 months. A chest radiograph reveals nodular densities, some with calcification, located mainly in the upper lobes. The microscopic appearance of a lung biopsy reveals granulomatous inflammation within the nodes, marked by the presence of giant cells with peripheral localization of nucleas and caseous necrosis in the central part of granuloma. What is the type of such granuloma? A. Granulomas with Virchow’s cells, B. Granulomas with Langhan’s cells, C. Granulomas with Mikulicz’s cells, D. Granulomas with foreign body cells, E. Interstitial inflammation

449. A 5yearold child reaches up to the stove and touches a pot of boiling soup. Within several hours, there is marked erythema of the skin of the fingers on the fight hand, and small blisters appear on the finger pads. Which of the following terms best describes this process? A. Fibrinous inflammation, B. Purulent inflammation, C. Ulceration, D. Serous inflammation, E. Granulomatous inflammation

450. Which of the following diseases can represents a constellation of syndromes having in common variable defects in both humoral and cellmediated immune responses as: marked lymphopenia with a deficiency of both T and B cells, normal numbers of B cells, which are nonfunctional owing to lack of Thelpers, normal numbers of circulating lymphocytes that bear the cell surface markers of very immature intra thymic T cells, thymus is hypoplastic and fetal in type, or it may be absent, lymph nodes are reduced in size? A. Severe Combined Immunodeficiency; B. DiGeorge's Syndrome; C. Bruton's Disease; D. Common Variable Immunodeficiency;

E. WiskottAldrich Syndrome

451. An 3yearold child was vulnerable to all forms of viral and bacterial infections, during clinical investigation it was revealed that he has had a normal thymus. He was successfully treated by transplantation of normal histocompatible bone marrow cells. Which of the following disease took place in patient? A. Severe Combined Immunodeficiency; B. DiGeorge's Syndrome; C. Bruton's Disease; D. Common Variable Immunodeficiency; E. WiskottAldrich Syndrome

452. Acquired inadequacy of immune answer because of fatigue or damage of the normally formed immune system is named as:A. Primary immunodeficiency syndrome; B. Secondary immunodeficiency syndrome; C. Autoimmune specific; D. Autoimmune nonspecific; E. Infectiousallergic

453. During autopsy it was found out firm in consistency spleen, on the cut surface it was pale and gray. The deposits of amyloid affect principally the splenic sinuses and extend to the splenic pulp, forming large, sheetlike deposits. What is the name of such spleen? A. Big white amyloid spleen; B. Goose spleen; C. Sago spleen; D. Small white amyloid spleen; E. Lardaceous spleen

454. A 27yearold male is on a scuba diving trip to the Caribbean. After descending to 50 m in the Blue Hole, he returns to the boat. About an hour later, he develops severe, painful myalgias and arthralgias. These symptoms abate over the next day. His symptoms are most likely the result of : A. Disseminated intravascular coagulation, B. Systemic vasodilatation, C. Venous thrombosis, D. Tissue nitrogen emboli, E. Fat globules in arterioles

455. The left breast of a 39yearold female is slightly enlarged compared with the right. The skin overlying this breast is thickened, reddishorange, and pitted. Mammography reveals a 3cm underlying density. A fineneedle aspirate of this mass reveals carcinoma. How is the gross appearance of the left breast best explained? A. Venous thrombosis, B. Lymphatic obstruction, C. Ischemia, D. Chronic passive congestion, E. Chronic inflammation

456. A 70yearold male is ambulating for the first time in 3 weeks after hospitalization for a cerebral infarction. Within minutes of returning to his hospital room, he has a sudden onset of dyspnea. He cannot be resuscitated. At autopsy, the gross appearance of the left lung is big and red. Which of the following risk factors contributed the most to this finding? A. Venous stasis, B. Pulmonary arterial atherosclerosis,

Page 27: Tests General 08

C. Chronic inflammation, D. Bronchopneumonia, E. Ischemia

457. A 76yearold female fell and fractured her left femoral trochanter. After 2 weeks in the hospital, her left leg is swollen, particularly her lower leg below the knee. She experiences pain on movement of this leg, and there is tenderness to palpation. A venogram reveals thrombosis of deep left leg veins. Which of the following complications is most likely to occur after these events? A. Gangrenous necrosis of the foot, B. Hematoma of the thigh, C. Disseminated intravascular coagulation, D. Pulmonary thromboembolism, E. Fat embolism

458. 55yearold female has had discomfort with swelling of her left leg for the past week. The leg is slightly difficult to move, but there is no pain on palpation. A venogram reveals thrombosis of deep left leg veins. Which of the following complications is most likely to occur in this patient? A. Gangrenous necrosis of the foot, B. Hematoma of the thigh, C. Disseminated intravascular coagulation, D. Pulmonary thromboembolism, E. Fat embolism

459. The author of «Cellular Pathology» is: A – K. Rokitansky, B – R.Virchow, C – A. Beily, D – J. Morgagni, E – T.Schwann

460. Choose the one which is not molecularmorphological display of cell organelles damage, caused by hypoxia: A – nucleus swelling and nucleolus disorganisation, B – destruction and fragmentation of mitochondria’s crystae, C – increasing of primary lisosomes’ amount, D – decreasing of primary lisosomes’ quantity, E – desagregation of ribosomes and polysomes

461. What reasons may lead to pathological changes and violation of cell organelles? A – hypoxia, B – intoxications, C – viral infections, D – tumor process, E – everything is right

462. Choose the one which is not a storage disease: A – Pompe’s disease, B – Tay – Sachs disease, C – Downsyndrome, D – Gaucher’s disease, E – NiemannPick disease

463. It was found that 8thmonths baby had an enormous hepatomegaly, hypoglycaemia, glycogen accumulations in tissues and organs. What storage disease may be supposed? A – Gaucher’s disease, B – Gierke’s disease, C – phenylketonuria, D – mucopolysacchaidoses,

E – Tay – Sachs disease

464. Choose the one, which is not included into group of chromosomes’ abnormalities : A – Klinefelter’s syndrome, B – Downsyndrome, C – Patau syndrome, D – DoubinJhonson syndrome, E – Turner syndrome

465. Hereditary albinism develops due to: A – deficiency of glucose6phosphatase, B – tyrosinase deficiency, C – excess of glycogen in tissues, D – phenylalanine hydroxylase deficiency, E – uric acid excess in blood

466. Disturbance of nucleoproteins’ metabolism with abundant production of uric acid and its salts leads to development of: A – albinism, B – melanoderma, C – gout, D – Gilbert disease, E – phenylketonuria

467. Porphyria is characterized by: A – disturbance of porphyrin metabolism, B – increased amount of porphyrin in blood, C – porphyrinuria, D – hypersensivity to ultraviolet radiation, E – everything is right

468. Choose the one which may lead to developed (secondary) porphyria : A – pellagra, B – cardiovascular insufficiency, C – disturbance of nucleoproteins’ metabolism, D – disturbance of lipopigments’ metabolism, E – generalized hemosiderosis

469. Young man 25years old has a permanent jaundice of skin, sclera and mucosa, periodical dyspeptic syndrome and hepatomegaly, increased levels of conjugated and nonconjugated bilirubin in blood, while another markers of liver function stay in norm. What hereditary disease may be supposed? A – Pompe’s disease, B – Wilson’s disease, C – DoubinJhonson disease, D – hereditary albinism, E – gout

470. WilsonConovalov disease develops due to disturbance of: A – porphyrin metabolism, B – copper metabolism, C – iron metabolism, D – melanin metabolism, C – bilirubin metabolism

471. Choose the feature of WilsonConovalov disease: A – glycogen accumulation in tissues and organs. B – KayserFleischer rings, C – leukoderma, D – uric acid excess in blood and tissues, E – porphyrinemia

Page 28: Tests General 08

472. Choose incorrect statement according to WilsonConovalov disease: A – it is a hereditary disease, B – it is characterized by copper accumulations in organs and tissues, C – there are hepatic and lenticular forms, D – there are cerebral and renal forms, E – outcome is unfavorable

473. Choose the one which is not included into group of pigmental hepatoses:A – DoubinJhonson syndrome, B – CriglerNajjar syndrome, C – Tay – Sachs disease, D – Gilbert syndrome, E – Rotor syndrome

474. Choose incorrect statement according to primary hemochromatosis : A – it is found bronze color of skin, B – it is accompanied by generalized hypomelanosis, C –bronze diabetes develops, D – pigment cirrhosis of liver develops, E – pigment cardiomyopathia and cardiovascular insufficiency may develop

475. A fight carotid endarterectomy is performed on a 69yearold female. Examination of the curetted atheromatous plaque reveals a grossly yellowtan, firm appearance. Microscopically, which of the following materials can be found in abundance in the form of crystals producing long, cleftlike spaces? A – Glycogen, B – Lipofuscin, C – Hemosiderin, D – Immunoglobulin, E Cholesterol

476. A 40yearold female with chronic congestive heart failure has a cough productive of rustcolored sputum. A sputum cytology specimen shows numerous hemosiderinladen macrophages. Which of the following subcellular structures in macrophages is most important for the accumulation of this pigment? A – Lysosome, B Endoplasmic reticulum, C – Ribosome, D Golgi apparatus, E Chromosome

477. A 55yearold female has dementia worsening over the past year. She also has avitaminosis and red, scaling skin in sunexposed areas. A watery diarrhea has been present for the past month. A – Beriberi, B – Cheilosis, C Diabetes mellitus, D – Pellagra, E Anemia

478. A 50yearold male with a history of chronic alcoholism has had increasing congestive heart failure for the past year. For the past month, he has experienced liver insufficiency and diabetes mellitus. The level of ferritin in blood is increased. These findings point to a pigment pathology as:A – Hemosiderosis,

B – Hemochromatosis, C – Hematin, D – Billirubin, E Melanin

479. A 25yearold male with panctopenia has had increasing pain in bones and pathologic fractures. There was observed storage of glycocerebrosides in spleen cells, liver, bone marrow and lymph nodules. Which of the following disease is most likely to observe? A – Gaucher’s disease type I, B Gaucher’s disease type II, C Gaucher’s disease type III, D NiemannPick disease, E Gilbert disease

480. A 37yearold male had painful muscle crump, especially after exercise. For the past week, he has had myoglobinuria. Which of the following disease is most likely to observe? A – Gaucher’s disease type I, B – Mc Ardle’s disease type IV, C Gaucher’s disease type III, D NiemannPick disease, E DoubinJhonson disease

481. At 43yearold male with a history of chronic hepatitis was observed greenbrown ring on the margin of the cornea. During laboratory investigation it was observed increased levels of serum copper, urine and hepatic copper, at decreased levels of serum ceruloplasmin. What disease do you think about? A – Pompe’s disease, B – Wilson’s disease, C – DoubinJhonson disease, D – hereditary albinism, E – Gout

482. During the induction of immediate hypersensitivity response, which of the following cells secretes cytokines that stimulate IgE production by B cells, promotes mast cell growth, and recruits and activates eosinophils? A. CD4+ lymphocytes, B. Natural killer (NK) cells, C. Macrophages, D. Dendritic cells, E. Neutrophils

483. A 45year old female with Graves’s disease during last two years has had increased level of thyroid hormones and AB against the thyroidstimulating hormone receptor. Which of the following types of hypersensitivity occurred in patient?A. The type I, B. The type II, C. The type III, D. The type IV, E. The type V

484. A young male walks into an elevator full of people who are coughing and sneezing, all of whom appear to have colds or the flu. The influenza viral particles that he inhales attach to respiratory epithelium, and viral transformation reduces the class I major histocompatibility complex molecules on these epithelial cells. Which of the following cells then responds to destroy the infected cells? A. Macrophage,

Page 29: Tests General 08

B, Neutrophil, C. CD4 cell, D. NK cell, E. Dendritic cell

485. A laboratory worker who was known to be "allergic" to fungal spores was accidentally exposed to culture of the incriminating fungus on a Friday afternoon. Within 60 minutes, he developed bouts of sneezing, watering of eyes, and nasal discharge. The next morning, while he was still at home, his symptoms reappeared, although the laboratory fungus was not present in his environment. His symptoms persisted through the weekend, and he went to see a doctor on Monday morning. If the physician examines a nasal discharge under the microscope, which of the following cells are likely to be seen? A. Mast cells and neutrophils, B. Lymphocytes and macrophages, C. Neutrophils, eosinophils, and CD4+ lymphocytes, D. Neutrophils and CDf+ lymphocytes, E. Mast cells, lymphocytes, and macrophages

486. In response to infection with Mycobacterium tuberculosis, a granuloma forms in the lung. Within the granuloma are cells expressing class II MHC antigens. These cells elaborate cytokines that promote fibroblastic production of collagen within the granuloma. From which of the following peripheral blood leukocytes are these class II antigenbearing cells derived? A. Neutrophil, B. Monocyte, C. B cell, D. NK cell, E. Basophil

487. A 15yearold female has the sudden onset of difficulty breathing within minutes after a bee sting. She also manifests marked urticaria, and there is marked edema of the hand that was stung. What type of hypersensitivity occurred in patient? A. The type I, B. The type II, C. The type III, D. The type IV, E. The type V

488. The 35yearold female was admitted into the hospital with nephrolithiasis. Anaphylactic shock was appeared after intravenous injection of contrast medium. Which cells take part in development of this reaction? A. Fibroblasts, B. Epitheliocytes, C. Myocytes, D. Tissue’s basophils, E. Neutrophils

489. The 29yearold male was admitted into the hospital with appendicitis. Anaphylactic shock was appeared after intravenous injection of anesthetic. What type of hypersensitivity occurred in patient? A. The type I, B. The type II, C. The type III, D. The type IV, E. The type V

490. At biopsy of lymph node it was found out a lot of

lymphoid follicles with large reproduction centers, increased number of mitoses. Which of the following processes is characterized by such morphologic changes? A. Atrophy of lymphoid tissue, B. Lymphosarcoma, C. Antigenic stimulation with follicular hyperplasia, D. Hodgkin’s disease, E. Metastases of cancer

491. At Myasthenia gravis AB react with acetylcholine receptors in the motor endplates of skeletal muscles impair neuromuscular transmission and therefore cause muscle weakness that can be seen at patient. What is the type of immune mechanism of the disease development?A. Complement and Fc receptormediated inflammation, B. Antibodymediated cellular dysfunction, C. Opsonization and phagocytosis, D. Tcell mediated cytolysis, E. Deposition of antigenantibody complexes and complement activation

492. At biopsy of stomach in patient with autoimmune gastritis it was found out: infiltration by lymphocytes and macrophages in mucous layer. Which type of the hypersensitivity reactions is connected with these morphologic changes? A. The type I, B. The type II, C. The type III, D. The type IV, E. The type V

493. A 9yearold boy has a sore throat. A throat culture grows group A hemolytic streptococcus. 17 days later develops darkcolored urine. A renal biopsy was performed. On immunefluoresence staining the biopsy shows granular deposition of IgG and complement around glomerular capillary loops. Which of the following immune hypersensitivity mechanisms is most likely responsible for this pattern of findings? A. The type I, B. The type II, C. The type III, D. The type IV, E. The type V

494. A 48yearold man has had a chronic cough with fever for 2 months. On physical examination his temperature is 37.9 C. A chest radiograph reveals a diffuse bilateral reticulonodular pattern. A transbronchial biopsy is performed. On microscopic examination of the biopsy there are focal areas of inflammation containing epithelioid macrophages, Langhans giant cells, and lymphocytes. These findings are most typical for which of the following immunologic responses? A. The type I, B. The type II, C. The type III, D. The type IV, E. The type V

495. Twelve hours after going on a hike through dense foliage, a 40yearold man notices a slightly raised and tender irregular reddish rash on one forearm that was not covered by clothing. This rash gradually increases in intensity for 2 days and then fades after two weeks. Which of the following forms of hypersensitivity is most likely demonstrated in this patient? A. The type I, B. The type II, C. The type III,

Page 30: Tests General 08

D. The type IV, E. The type V

496. Approximately 10 days after AG administration, at 25year patient clinical symptoms as urticaria, arthralgias, lymph node enlargement and proteinuria were revealed. Which of the following forms of hypersensitivity is most likely demonstrated in this patient? A. The type I, B. The type II, C. The type III, D. The type IV, E. The type V

497. In 60yearold patient, during examination the cancer of a prostate gland with metastases in the lower parts of the column, pelvic bones and pathological fracture of a femur was found out. What pathological process is a basis of the development of metastases? A Tissue embolism, B Bacterial embolism, C Airembolism, D Embolism by foreign matters, E Necrosis of tissue

498. The left breast of a 39yearold female is slightly enlarged compared with the right. The skin overlying this breast is thickened, reddishorange, and pitted. Mammography reveals a 3cm underlying density. A fineneedle aspirate of this mass reveals carcinoma. How is the gross appearance of the left breast best explained? A Venous thrombosis, B Lymphatic obstrnction, C – lschemia, D Chronic passive congestion, E Chronic inflammation

499. During autopsy of 46yearold patient, died because brain hemorrhage, it was found out liver cirrhosis, degenerative symmetrical changers in the area of lens nuclei, caudal body, pale globe and cortex of brain. Such kind of injury is known as: A – Pompe’s disease, B – Wilson’s disease, C – DoubinJhonson disease, D – hereditary albinism, E – Gout

500. In 67yearold female, with increased levels of urine and uric acid in blood, deformation of small joints of feet and hands were observed. During biopsy investigations of joints it was observed accumulation of salts in zones of tissue necrosis with agranulomatous and giant cells reaction, proliferation of connective tissue with formation of nodules. Which of the following disease is more possible? A – Pompe’s disease, B – Wilson’s disease, C – DoubinJhonson disease, D – hereditary albinism, E – Gout

501. Deposition of cystine can cause proximal renal tubular acidosis. Such station is most likely to occur at: A – DoubinJhonson syndrome, B – CriglerNajjar syndrome, C – Fanconi syndrome, D – Gilbert syndrome,

E – Rotor syndrome

502. A patient with severe fracture of both femoral bones has died in a 4 hours after trauma. What is the cause of patient’s death and what kind of additional staining of microscopic sections is it necessary to perform in order to prove the diagnose? A The lungs and brain by Sudan III, B The lungs and kidney by Congored, C The lungs and liver by Perl’s reaction, D The brain and kidney by Congored, E The kidney and heart by toluidinblue

503. During examination of 9yearold patient with cystine nephropathy it was observed rickets, growth retardation, patchy retinal depigmentation, sparkling crystals in cornea and abundant crystals in conjunctiva. Which of the following storage disease is observed? A – Phenylketonuria, B – Cystinosis, C Pompe’s disease, D NiemannPick disease, E NormanLanding disease

504. In 60yearold patient, during examination the cancer of a prostate gland with metastases in the lower parts of the column, pelvic bones and pathological fracture of a femur was found out. What pathological process is a basis of the development of metastases? A. Tissue embolism, B. Bacterial embolism, C. Airembolism, D. Embolism by foreign matters, E. Necrosis of tissue

505. A patient with severe fracture of both femoral bones has died in a 4 hours after trauma. What is the cause of patient’s death and what kind of additional staining of microscopic sections is it necessary to perform in order to prove the diagnose? A. The lungs and brain by Sudan III, B. The lungs and kidney by Congored, C. The lungs and liver by Perl’s reaction, D. The brain and kidney by Congored, E. The kidney and heart by toluidinblue

506. At microscopic research of the blood clot removed from the vein, it was revealed, that blood clot is substituted by connective tissue with zones of cracks and formation of ducts covered by endothelial cells. Identify type of thrombosis outcome: A. Organization and recanalization of thrombus, B. Aseptic autolysis, C. Transformation into thromboemboli, D. Septic autolysis, E. Calcification of blood clot

507. A 54yearold patient died at septic shock with expressed DICsyndrome. What are the most specific morphological changers characterized DIC syndrome can be found out by pathologist at autopsy: A. Plural ischemic microinfarctions in brain, lungs, kidneys due to fibrin thrombi in small arteries, B. Blood clots in large vessels, C. Dilatation blood clot in the cavity of right auricle, D. Hemosiderosis of lungs, E. Generalized lymphadenopathy

Page 31: Tests General 08

508. A 35yearold woman died because of obstetric complications accompanied by polyorgan insufficiency. During autopsy it was revealed numerous blood clods in small arteries of brain, heart, lungs, kidneys, and hearth of hemorrhage in mucous and serous membranes of organs. What process is exposed in this case? A. Disseminated intravascular coagulation, B. Systemic vasodilatation, C. Venous thrombosis, D. Disseminated stasis, E. Disseminated acute ischemia

509. Diver died 2 hours after quick decompression. At autopsy it was revealed crepitating of skin, anemia of internal organs, foamy blood in the cavities of heart and large vessels. Microscopically numerous emboli were found in the capillaries of brain, liver and kidneys, and also numerous foci of ischemia and necrosis in organs; lungs were swollen, with hemorrhage. Identify, what embolism was exposed in this case? A. Gas, B. Tissue, C. Air, D. Fatty, E. By foreign bodies

510. A 27yearold man hurt veins of the neck; he died suddenly because of acute respiratory insufficiency. At autopsy it was found out foamy blood in the cavities of right parts of the heart and large vessels foamy. Microscopically numerous emboli were found out in the pulmonary arteries and capillaries. Identify, what embolism was exposed in this case? A. Air. B. Tissue. C. Gas. D. Thromboembolism. E. Fatty

511. At autopsy during investigation of lungs it was found out numerous hemorrhagic infarcts and dense brown color masses in vessels. Veins of lower extremities are varicose extended with darkbrown color masses inside. What pathological process is described? A. Thromboembolism of pulmonary arteries. B. Fatty embolism of pulmonary artery. C. Tissue embolism of pulmonary artery. D. Brown induration of lungs. E. Hemorragic infiltration

512. During histological investigation of soft tissue it was found out closed by mass of blood clot vessel. Identify type of blood clot according to the vessel wall’s relation: A. Nearwall, B. Obstructive, C. Dilatational, D. Axial, E. Spherical

513. Main factors that participate in thrombus formation are: A. All include, B. Endothelial integrity, C. Abnormal blood flow, D. Hupercoagulability, E. Everything is wrong

514. 46yearold woman has suffered rheumatic fever with combined mitral heart disease. She died because of pneumocardial insufficiency. On dissection there was found out brown induration of lungs. Name the type of violation of blood circulation. A. Chronic left ventricle insufficiency. B. Chronic right ventricle insufficiency. C. Acute left ventricle insufficiency. D. Acute right ventricle insufficiency E. Portal гипертензия.

515. During endoscopical investigation of stomach at 32yearold patient it was found out chronic ulcer with clear edges and brownblack bottom, the signs of the gastric bleeding were present. Identify the mechanism of development of gastric bleeding. A. Diapedesis B. Spasm C. Corrosion of the vessel wall D. Rupture of the vessel wall E. Increased permeability

516. At 45yearold patient, who suffered rheumatic fever with combined mitral heart disease and dying of heart decompensation, at autopsy many pointlike hemorrhages were found out in brain tissue. Identify the mechanism of development of hemorrhages? A. Increased permeability. B. Spasm. C. Rupture of the vessel wall. D. Swollening of the vessel wall. E. Corrosion of the vessel wall.

517. Intracellular accumulation of proteins in liver cells named:A Mellory bodies; B Russell bodies; C Keratin bodies; D Cellular swelling; E “Goose” liver

518. A collapse and hyperemia of peritoneum developed at a patient after deleting of 12 liters of ascytic liquid from abdominal cavity. What is the type of arterial hyperemia of peritoneum? A. Vacant hyperemia, B. Inflammatory hyperemia, C. Hyperemia after anemia D. Collateral hyperemia E. Angioneurotic hyperemia

519. While shaving one morning, a 23yearold male nicks his upper lip with the razor. Within a second after this injury, blood loss from a small dermal arteriole is reduced through: A Activated protein C. B Vasoconstriction. C Platelet aggregation. D Neutrophil chemotaxis. E Fibrin polymerization

520. Shock can be characterized as: A. Clinical state of cardiovascular collapse that develops under the influence of extrapowerful stimuli B. Acute disorders of blood circulation during acute arrhythmia and myocardial insufficiency.

Page 32: Tests General 08

C. Chronic disorders of blood circulation during acute arrhythmia and myocardial insufficiency. D. Morphological changers at cardiovascular collapse that develops under the influence of extrapowerful stimuli. E. Local acute disorders of blood circulation at trauma.

521. Necrosis of organ it is: A. It is programmed cell’s death in the living organism with fragmentation of cell into apoptotic bodies at the end the process B. It is the premature death and destruction of cell’s organelles in the living organism C. It is cell’s death in the living organism under action of critical damage factors D. It is postmortem destruction of cells under action of the bacteria’s enzymes E. It is destruction of all components of organ under action of critical alteration

522. A 3cm, right middle lobe lung nodule was seen on a chest radiograph of an asymptomatic 37yearold male. The nodule was excised with a pulmonary wedge resection by the thoracic surgeon. On sectioning by the pathologist, the nodule was sharply circumscribed and had a soft, white center. Culture of tissue from the nodule grew Mycobacterium tuberculosis.A. Fat necrosis B. Caseous necrosis C. Liquefactive necrosis D. Gangrenous necrosis E. Coagulative necrosis

523. During an operation in spleen it was seen wedgeshaped whiteyellowish color foci of necrosis with a base on the organ capsule. The cause of this process is thrombosis of arteries. Identify the pathological process in organ: A. Ischemic infarction B. Hemorrhagic infarction C. White infarction with hemorrhagic halo D. Gangrine E. Bedsores

524. At patient with syphilis an areas of skin depigmentation appeared on the upper parts of the back. What pathological process is observed on the skin? А. Parakeratosis В. Metaplasia С. Leukoplakia D. Dysplasia Е. Leukoderma

525. Surgical operation was performed at patient with brain tumor. At histological investigation it was revealed roundform tumor that is consists of cells with intracellular accumulation of brown pigment, Prussian blue reaction is negative? What kind of pigment accumulates in tumor cells? А. Hematoidin В. Melanin C. Hemosiderin D. Porfirin Е. Haematin

526. During endoscopical investigation of stomach at 32yearold patient it was found out acute ulcer with clear edges and brownblack bottom. What pigment is formed in the bottom of ulcer? А. Ferritin В. Melanin C. Porfirin

D. Bilirubin Е. Haematin hydrochlorid

527. At histologic examination of tuberculoma resected from right lung a necrosis in center is found out. Call a kind of the necrosis. A. Fatty B. Liquefactive C. Caseous D. Coagulative E. Gangrene

528. Acute pain at the foot appeared in elderly patient with the atherosclerosis of lower extremities. Left extremity became soft and was increased in volume, skin became blackbrown color with maceration. The demarcation area is absent. Your diagnosis: A Moist gangrene B Mummification of foot C Coagulate necrosis of foot. D Dry gangrene E Sequester of the foot

529. Intracellular fatty vacuoles at “goose” liver are colored in orangered color by staining:A – hematoxylin at eosin staining; B – Sudan III, C – VanGison; D – PAS reaction; E – Prussian blue

530. During surgical operation in patient with stomach profuse haemorrhage it was found a tumor growth. What method of urgent study allows to diagnose the character of pathological process?A – endoscopic biopsyB – express biopsy (intraoperative)C – puncture biopsyD – aspiration biopsyE – excision biopsy

531. What pathomorphological method doesn’t allow investigation of morphological changes at molecularsubcellular level?A – light microscopy;B – electronic microscopy;C – immunohistochemical analysisD – autoradiographyE – cytochemical analysis

532. Pathoanatomical diagnosis includes:A – main disease;B – concomitant diseases;C – complications of disease;D – everything is rightE – everything is wrong

533.Pathoanatomical diagnosis begins from:A main disease that has leaded to death;B – immediate cause that has leaded to death;C – concomitant diseasesD – clinical diagnosisE – complication of disease that has leaded to death